Download as pdf or txt
Download as pdf or txt
You are on page 1of 88

MINISTRY OF PUBLIC HEALTH OF UKRAINE

IVANO-FRANKIVSK NATIONAL MEDICAL


UNIVERSITY

STEP - 1
PATHOPHYSIOLOGY

FOR STUDENTS
OF MEDICAL FACULTY

Gerasymchuk M.R.
Pobigun A.A.
Cherkasova V.V.

IVANO-FRANKIVSK
Gerasymchuk M.R. STEP-1 PATHOPHYSIOLOGY for Students of
Medical faculty / M.R. Gearsymchuk, A.A. Pobigun, V.V. Cherkasova //
IFNMU. Department of pathophysiology. – 2012. – 88 p.

Disscussed and approved on prophil commission of medical&biological


disciplines meeting of Ivano-Frankivsk National Medical University.
Protocol № ___ from «____» ___________ 2012 year

2
CONTENT

1. Influence External Factors 5


2. Pathology of hereditary & constitution 5
3. Enzymopathies 11
4. Pathology of Immune Reactivity 13
5. Allergy 16
6. Pathology of Macro- & Microcirculation 20
7. Inflammation 24
8. Pathophysiology of Tumor Growth 27
9. Cell injury 28
10. Starvation & Pathology of Metabolism 30
11. Disorders of Carbohydrate Metabolism 31
12. Disorders of Lipid Metabolism 34
13. Disorders of Water-Electrolite Balance 34
14. Disorders of Acid-Base Balance 38
15. Vitamine Disorders 41
16. Disorders of termoregulation. Fever 44
17. Hipoxia 46
18. Disorders of Respiratory System 47
19. Pathophysiology of Blood. Disorders of RBC. 50
20. Pathophysiology of Blood. Disorders of WBC. 55
21. Disorders of Haemostasis 57
22. Disorders Cardiovascular System 60
23. Disorders of Digestive System 67
24. Liver pathology 70
25. Kidneys pathology 72
26. Disorders of Endocrine System 76
27. Disorders of CNS 82
28. Answers to the Tests 89
29. Recommended Literature 90

3
4
important factor of syndrome
1. Influence of External pathogenesis?
factors A. Increased activity of
anticoagulative system factors
B. Vascular wall damage
1. In result of the damage of one of
C. Thrombocytopenia
the Atomic Power Plant reacttor the
D. Decreased activity of coagulative
run-out of radioelements happened.
factors
People in the increased radiation zone
E. Increased activity of fibrinolysis
were radiated with approximately 250-
factors
300 r. They were immediately
hospitalized. What changes in the
blood count would be typical? 2. Pathology of
A. Neutropenia hereditary & constitution
B. Leukopenia
C. Anemia
5. While studing of the family tree
D. Thrombopenia
with history of hypertrichosis
E. Lymphopenia
(hyperhirsutism of the ear) this sign
was founded only in the men and it was
2. The preventive radioprotector
inherited from father to the son. Define
was given to a worker of a nuclear
the type of hypertrichosis inheritance?
power station. What mechanism from
A. Autosomal-dominant
the below mentioned is considered to
B. Autosomal- recessive
be the main mechanism of
C. Connected with Y-chromosome
radioprotection?
D. Connected with Х-chromosome
A. Increasing of respiration
dominant
B. Inhibition of free radicals
E. Connected with Х-chromosome
formation
recessive
C. Activation of oxidation reactions
D. Prevention of tissue’s hypoxia
6. 46 chromosomes were revealed
E. Increasing of tissue blood supply
on karyotype examination of the 5-year-
old girl. One of the 15th pair of
3. The action of electric current on
chromosomes is longer than usual due
the exitable cell caused depolarization
to connected chromosome from the 21
of its membrane. Movement of what
pair. What type of mutation does this
ions through the membrane caused
girl have?
depolarisation?
A. Deletion
A. Сl-
B. Insufficiency
B. Са2+
C. Inversion
C. Na+
D. Duplication
D. НСО3-
E. Translocation
E. К+
7. Daltonism was diagnosed in a 7-
4. A disaster fighter at a nuclear
year-old boy while prophylactic medical
power plant developed hemorrhagic
examination. Parents are healthy, color
syndrome on the background of acute
vision is normal. Grandfather from the
radiation disease. What is the most
mother's side has the same disorder.
What is the type of inheriting of this inherited by son from his father. What is
anomaly? the type of hypertrichosis inheritance?
A. Autosomal-recessive A. Y-linked chromosome
B. Recessive, sex-linked B. X-linked recessive chromosome
C. Incomplete domination C. Autosome-recessive
D. Autosomal-dominant D. Autosome-dominant
E. Dominant, connected with sex E. X-linked dominant chromosome

8. Part of the DNA chain turned 12. After the genealogy


about 180 degree due to gamma analysis a geneticist came to the
radiation. What type of mutation took conclusion: a feature is manifested in
place in the DNA chain? each generation, men and women
A. Replication inherit the feature with equal frequency,
B. Translocation parents in the equal way give this
C. Inversion feature to their offspring. What type of
D. Doubling inheritance does the investigated
E. Deletion feature have?
A. Autosomal-dominant
9. White-haired, with blue eyes girl B. Х-linked dominant inheritance
was born in healthy parents. Irritability, C. Х-linked recessive inheritance
anxiety, troubled sleep and feeding D. Y-linked inheritance
developed in the first months of life of E. Autosomal-recessive
the infant. What method of genetic
investigation should be used for the 13. Woman applied to the medico-
exact diagnosis? genetic consulting centre for
A. Twin information about the risk of
B. Cytological haemophilia in her son. Her husband
C. Population-statistical has been suffering from this disease
D. Biochemical since birth. Woman and her parents are
E. Genealogical healthy (don't have haemophilia). Is the
boy likely to have the disease in this
10. A 40-year-old pregnant woman family?
underwent amniocentesis. The A. 75% of the boys will be ill
examination of fetus karyotype B. 25% of the boys will be ill
revealed 47ХУ+21. What pathology of C. All boys will be ill
the fetus was found out? D. 50% of the boys will be ill
A. Schereschevsky-Terner's E. All boys will be healthy
disease
B. Patau's disease 14. The examination of an
C. Phenylketonuria youth with mental retardation revealed
D. Down's syndrome eunuchoid body construction and
E. Klinefelter's syndrome genitals underdevelopment. The cells
of the oral cavity cantained chromatine.
11. The study of the genealogy of a What method of genetic investigation
family with hypertrichosis (helix should be performed to make more
excessive pilosis) demonstrated, that specified diagnosis?
this feature is manifested in all A. Biochemical
generations only in men and is B. Dermatoglyphics

6
C. Population-statistic there were no cases of hemophilia in
D. Clinico-genealogical her family. What is the risk of having a
E. Cytological sick child in this family?
A. 25%
15. An 18-year-old man with B. 0
asthenic body constitution: tall, narrow C. 100%
shoulders, broad pelvis and with poor D. 75%
hair on his face came to the geneticist. E. 50%
There was marked mental retardatoin.
The preliminary diagnosis was 19 A woman who was sick with
Klinefelter's syndrome. What method rubella during the pregnancy gave birth
of medical genetics can confirm the to a deaf child with hare lip and cleft
diagnosis? palate. This congenital defect is an
A. Population-statistic example of:
B. Dermatoglyphics A. Genocopy
C. Cytogenic B. Down’s syndrome
D. Gemellary C. Edward’s syndrome
E. Genealogic D. Patau’s syndrome
E. Phenocopy
16. The study of the genealogy of a
family with hypertrichosis (helix 20. A patient, who suffers from
excessive pilosis) has demonstrated congenital erythropoietic porphyria, has
that this symptom is manifested in all skin photosensitivity. The accumulation
generations only in men and is of what compound in the skin can
inherited by son from his father. What cause it?
is the type of hypertrichosis A. Uroporphyrinogen 1
inheritance? B. Uroporphyrinogen 2
A. X-linked recessive chromosome C. Protoporphyrin
B. Autosome-recessive D. Coproporphyrinogen 3
C. X-linked dominant chromosome E. Heme
D. Y-linked chromosome
E. Autosome-dominant 21. An individual is characterized by
rounded face, broad forehead, a
17. A healthy woman has three mongolian type of eyelid fold, flattened
sons affected by color blindness who nasal bridge, permanently open mouth,
were born after her two marriages. projecting lower lip, protruding tongue,
Children both of her husbands are short neck, flat hands, and stubby
healthy. What is the most possible fingers. What diagnosis can be put to
pattern of inheritance of this disease? the patient?
A. X-linked recessive A. Down's syndrome
B. Autosomal recessive B. Turner's syndrome
C. Y-linked C. Klinefelter's syndrome
D. Autosomal dominant D. Supermales
E. X-linked dominant E. Alkaptonuria

18 A couple came for medical 22. Hypertrychosis of auricles is


genetic counseling. The man has caused by a gene that is localized in Y-
hemophilia, the woman is healthy and chromosome. Father has this feature.

7
What is the probability to give birth to 26. Healthy parents have got a fair-
a boy with such anomaly? haired, blue-eyed girl. Irritability,
A. 25% anxiety, sleep and feeding disturbance
B. 0% developed in the first months of the
C. 35% infant's life. Neurological examination
D. 75% revealed developmental lag. What
E. 100% method of genetic investigation should
be used for the exact diagnosis?
23. Objective examination of a A. Cytological
patient revealed: slender figure, big B. Genealogical
skull, highly developed frontal region C. Biochemical
of face, short extremities. What D. Population-statistical
constitutional type is it characteristic E. Gemellary
for?
A. Respiratory 27. Analysis of amniotic fluid that
B. Mixed was obtained as a result of
C. Digestive amniocentesis (puncture of amniotic
D. Cerebral sac) revealed cells the nuclei of which
E. Muscular contain sex chromatin (Barr's body).
What can it be evidence of?
24. A 5-year-old child who often A. Development of male fetus
fells ill with respiratory diseases has B. Genetic disorders of fetus
eczematous appearances after development
consumption of some food products, C. Development of female fetus
tendency to prolonged course of D. Trisomy
inflammatory processes. What kind of E. Polyploidy
diathesis can be suspected in this
case? 28. A woman with 0 (I) bllod group
A. Lymphohypoplastic has born a child with AB blood group.
B. Hemmorhagic This woman's husband has A blood
C. Exudative-catharral group. What genetic interaction
D. Asthenic explains this phenomenon?
E. Arthritism A. Recessive epistasis
B. Codominance
25. A genetics specialist analyzed C. Polymery
the genealogy of a family and found D. Incomplete dominance
that both males and females may E. Complementation
have the illness, not across all the
generations, and that healthy parents 29. Healthy parents have got a fair-
may have ill children. What is the type haired, blue-eyed girl. Irritability,
of illness inheritance? anxiety, sleep and feeding disturbance
A. Y-linked developed in the first months of the
B. Autosomal recessive infant's life. Neurological examination
C. X-linked recessive revealed developmental lag. What
D. Autosomal dominant method of genetic investigation should
E. X-linked dominant be used for the exact diagnosis?
A. Cytological
B. Genealogical

8
C. Biochemical 33. As a result of prophylactic
D. Gemellary medical examination a 7 year old boy
E. Population-statistical was diagnosed with Lesch-Nyhan
syndrome (only boys fall ill with it). The
30. Examination of a patient boy's parents are healthy but his
revealed a strong, balanced, inert type grandfather by his mother's side suffers
of higher nervous activity according to from the same disease. What type of
Pavlov. What temperament type does disease inheritance is it?
the patient have (according to A. Dominant, sex-linked
Hippocrates classification)? B. Semidominance
A. Choleric C. Recessive, sex-linked
B. Melancholic D. Autosomal dominant
C. Sanguine E. Autosomal recessive
D. Phlegmatic
E. - 34. Continuous taking of some drugs
foregoing the pregnancy increase the
31. Autopsy of a newborn boy risk of giving birth to a child with genetic
revealed polydactylia, microcephalia, defects. What is this effect called?
cheiloschisis and uranoschisis as well A. Embryotoxic effect
as hypertrophy of parenchimatous B. Blastomogenic effect
organs. These defects correspond C. Mutagenic effect
with the description of Patau's D. Fetotoxical effect
syndrom e. What is the most probable E. Teratogenic effect
cause of this pathology?
A. Trisomy of the 13th 35. A couple had a child with Down's
chromosome disease. Mother is 42 years old. This
B. Partial monosomy disease is most probably caused by the
C. Trisomy of the 21st following impairment of prenatal
chromosome development:
D. Nondisjunction of sex A. Blastopathy
chromosomes B. Gametopathy
E. Trisomy of the 18th C. Embryopathy
chromosome D. Non-specific fetopathy
E. Specific fetopathy
32. A 32 y.o. man is tall, he has
gynecomastia, adult woman pattern of 36. Examination of a newborn boy's
hair distribution, high voice, mental genitalia revealed an urethral hiatus
deficiency, sterility. Provisional that opens on the undersite of his
diagnosis is Klinefelter's syndrom e. In penis. What malformation is it?
order to specify diagnosis it is A. Cryptorhidism
necessary to analize: B. Monorchism
A. Caryotype C. Epispadia
B. Blood group D. Hermaphroditism
C. Genealogy E. Hypospadias
D. Leukogram
E. Spermatogenesis 37. Cytogenetic examination of a
patient with reproductive dysfunction
revealed normal karyotype 46 ХY in

9
some cells, but most cells have B. Hemorrhagic diathesis
karyotype of Klinefelter's syndrome - C. Lymphohypoplastic diathesis
47 ХХY. Such cell heterogenity is D. Exudative diathesis
called: E. Gouty diathesis
A. Mosaicism
B. Duplication 41 Sex chromosomes of a woman
C. Inversion didn't separate and move to the
D. Monomorphism opposite poles of a cell during
E. Transposition gametogenesis (meiosis). The ovum
was impregnated with a normal
38. A child is pale, pastose, spermatozoon. Which chromosomal
muscular tissue is bad developed, disease can be found in her child?
lymph nodes are enlarged. He often A. Turner's syndrome
suffers from angina and pharyngitis, B. Cat cry syndrome
blood has signs of lymphocytosis. The C. Edwards' syndrome
child is also predisposed to D. Down's syndrome
autoallergic diseases. What type of E. Patau's syndrome
diathesis can be presumed in this
case? 42. Examination of a 12-year-old
A. Exudative boy with developmental lag revealed
B. Gouty achondroplasia: disproportional
C. Asthenic constitution with evident shortening of
D. Hemorrhagic upper and lower limbs as a result of
E. Lymphohypoplastic growth disorder of epiphyseal cartilages
of long tubal bones. This disease is:
39. A family of students who came A. Inherited, sex-linked
from Africa got a child with anemia B. Inherited, dominant
signs. The child died soon. C. Inherited, recessive
Examination revealed that the child's D. Congenital
erythrocytes have abnormal semilunar E. Acquired
shape. Specify genotypes of the
child's parents: 43. Medical examination at the
A. аа х аа military registration and enlistment
B. Аа х аа office revealed that a 15-year-old boy
C. Аа х АА was high, with eunuchoid body
D. АА х АА proportions, gynecomastia, female
E. Аа х Аа pattern of pubic hair distribution. The
boy had also fat deposits on the thighs,
40. A child with a history of no facial hair, high voice, subnormal
frequent angine and pharyngitis has intelligence quotient. Which karyotype
been diagnosed with corresponds with this disease?
lymphadenopathy and splenomegaly. A. 47, XXY
His appearance is characterised by B. 47, XXX
pastosity and paleness, muscular C. 45, XO
tissue is poorly developed. D. 46, XY
Lymphocytosis is present. What kind E. 46, XX
of diathesis is it?
A. Asthenic diathesis

10
44. A 28-year-old female patient C. Inherited insufficiency of catalase
consulted a gynecologist about D. Inherited insufficiency
sterility. Examination revealed erythrocyte's phosphatdehydrogenase
underdeveloped ovaries and uterus, E. Pus in the wound
irregular menstrual cycle. Analysis of
the sex chromatin revealed 2 Barr’s 48. A child has got galactosemia.
bodies in most somatic cells. What Concentration of glucose in blood has
chromosome disease is most likely? not considerably changed. Deficiency
A. Klinefelter's syndrome of what enzyme caused this illness?
B. Turner's syndrome A. Galactokinase
C. Edwards' syndrome B. Phosphoglucomutase
D. Patau's syndrome C. Amylo-1,6-glucosidase
E. Triple X syndrome D. Galactose-1-phosphate
uridyltransferase
45. One of the parents is suspected E. Hexokinase
of having phenylketonuria recessive
gene. What is the risk of giving birth to 49. While examining the child the
a child with inborn phenylketonuria? doctor revealed symmetric cheeks
A. 75% roughness, diarrhea, disfunction of the
B. 100% nervous system. Lack of what food
C. 0% components caused it?
D. 25% A. Lysine, ascorbic acid
E. 50% B. Threonine, pantothenic acid
C. Methionine, lipoic acid
3. Enzymopathies D. Nicotinic acid, tryptophane
E. Phenylalanine, pangamic acid

46. Patient experienced increased 50. A 13-year-old boy complains of


susceptibility of the skin to the general weakness, dizziness, tiredness.
sunlight. His urine after some time He is mentally retarded. Increased level
became dark-red. What is the most of valine, isoleucine, leucine is in the
likely cause of this? blood and urine. Urine has specific
A. Pellagra smell. What is the diagnosis?
B. Alkaptonuria A. Addison's disease
C. Porphyria B. Tyrosinosis
D. Albinism C. Histidinemia
E. Hemolytic jaundice D. Maple syrup urine disease
E. Graves' disease
47. A patient with abscess of the
cut wound applied to the 51. Albinos can't stand sun impact
traumatological department.The they don't aquire sun-tan but get
wound was washed with 3% hydrogen sunburns. Disturbed metabolism of
peroxide to be cleaned from the pus. what aminoacid underlies this
Foam was not observed. What caused phenomenon?
inefficiency of the drug? A. Phenilalanine
A. Low concentration H2O2 B. Histidine
B. Shallow wound C. Tryptophan
D. Methionine

11
E. Glutamic acid 56. A patient has been diagnosed
with alkaptonuria. Choose an enzyme
52. Blood of a 12 year old boy whose deficiency can be the reason for
presents low concentration of uric acid this pathology:
and accumulation of xanthine and A. Dioxyphenylalanine
hypoxanthine. This child has genetic decarboxylase
defect of the following enzyme: B. Homogentisic acid oxidase
A. Ornithine carbamoyltransferase C. Phenylalanine hydroxylase
B. Arginase D. Glutamate dehydrogenase
C. Xanthine oxidase E. Pyruvate dehydrogenase
D. Glycerylkinase
E. Urease 57. A child suffers from drug
idiosyncrasy. What is the cause of such
53. Examination of a patient reaction?
revealed typical presentations of A. Inhibition of microsomal liver
collagenosis. This pathology is enzymes
characterized by increase of the B. Exhaustion of substrate
following urine index: interacting with pharmaceutical
A. Arginine substance
B. Hydroxyproline C. Hereditary enzymopathy
C. Glucose D. Accumulation of pharmaceutical
D. Mineral salts substance
E. Ammonium salts E. Associated disease of target
organ
54. A newborn child suffers from
milk curdling in stomach, this means 58. A newborn develops dyspepsia
that soluble milk proteins (caseins) after the milk feeding. When the milk is
transform to insoluble proteins substituted by the glucose solution the
(paracaseins) by means of calcium dyspepsia symptoms disappear. The
ions and a certain enzyme What newborn has the subnormal activity of
enzyme takes part in this process? the following enzyme:
A. Pepsin A. Amylase
B. Renin B. Maltase
C. Lipase C. Invertase
D. Secretin D. Isomaltase
E. Gastrin E. Lactase

55. A full-term newborn child has 59. A 46-year-old female patient has
yellowish skin and mucous a continuous history of progressive
membranes. This might be probably muscular (Duchenne's) dystrophy.
caused by temporary deficiency of the Which blood enzyme changes will be of
following enzyme: diagnostic value in this case?
A. Biliverdin reductase A. Pyruvate dehydrogenase
B. Heme synthetase B. Lactate dehydrogenase
C. Uridine transferase C. Creatine phosphokinase
D. Heme oxygenase D. Glutamate dehydrogenase
E. UDPglucoronyltransferase E. Adenylate cyclase

12
60. A 6 year old child was delivered amount of macrophages, lymphocytes,
to a hospital. Examination revealed lymphatic follicles in the cortical layer
that the child couldn't fix his eyes, and large amount of plasma cells were
didn't keep his eyes on toys, eye revealed on histological examination.
ground had the cherry-red spot sign. What process in the lymphatic nodules
Laboratory analyses showed that represent these histological changes?
brain, liver and spleen had high rate of A. Tumour transformation
ganglioside glycometide. What B. Antigen stimulation
congenital disease is the child ill with? C. Innate insufficiency of the
A. Tay-Sachs disease lymphoid tissue
B. Turner's syndrome D. Hypersensibility reaction
C. Wilson's syndrome E. Acquired insufficiency of the
D. Niemann-Pick disease lymphoid tissue
E. MacArdle disease
64. A 12-year-old boy often suffers
4. Pathology of from virus and bacterial infections and
eczematous skin lesions. Enlargement
Immune Reactivity of T-lymphocytes and IgM with normal
IgA and IgG was revealed on
61. In a patient with clinical examination. What type of immune
signs of immunodeficiency the number system pathology is presented in the
and functional activity of T and B patient?
lymphocytes are not changed. Defect A. Bruton's hypogammaglobulinemia
with dysfunction of antigen- B. Hereditary immundeficiency of the
presentation to the immunocompetent complement system
cells was found during investigation C. Composite immunedefficiency
on the molecule level. Defect of what D. Hypoplasia of thymus
cells is the most probable? E. Turner's syndrome
A. Т-lymphocytes, В-lymphocytes
B. 0-lymphocytes 65. The specimens present sections
C. Fibroblasts, Т-lymphocytes, В- of haemopoetic and immunogenetic
lymphocytes organs. Organ has lymph tissue
D. NK-cells forming different structures (lymph
E. Macrophages, monocytes nodes,lobules, bars). In what organ
does antigen-independent proliferation
62. Live vaccine is injected into the and differantiation take place?
human body. Increasing activity of A. Thymus
what cells of connective tissue can be B. Lymphatic nodes
expected? C. Hemolymph nodes
A. Fibroblasts and labrocytes D. Tonsil
B. Adipocytes and adventitious E. Spleen
cells
C. Macrophages and fibroblasts 66. After the radiactive
D. Plasmocytes and lymphocytes exposure a patient has stem cells
E. Pigmentocytes and pericytes disorder. The regeneration of what cells
of friable connective tissue will be
63. Local lymphonodules enlarged damaged?
near the infected wound. Increased A. Macrophages

13
B. Pericytes 70. A patient with clinical
C. Fibroblasts presentations of immunodeficiency
D. Pigment cells went through immunological
E. Adipocytes examinations. They revealed
significant loss of cells that form
67. Blood analysis of a patient rosettes with erythrocytes of a ram.
showed signs of HIV infection (human What conclusion can be made
immunodeficiency virus). Which cells according to the analysis data?
does HIV-virus primarily A. Decrease of complement system
A. Specialized nervous cells rate
(neurons) B. Decrease of B-lymphocytes rate
B. Mast cells C. Insufficiency of effector cells of
C. Cells that contain receptor IgM humoral immunity
(B-lymphocytes) D. Decrease of T-lymphocytes rate
D. Proliferating cells (stem E. Decrease of natural killer cell rate
hematoplastic cells)
E. Cells that contain receptor T4 71. For the purpose of myocardium
(T-helpers) infarction treatment a patient was
injected with embryonal stem cells
68. A patient with clinical signs of derived from this very patient by means
immunodeficiency has unchanged of therapeutic cloning. What
number and functional activity of T transplantation type is it?
and B lymphocytes. Dysfunction's A. Allotransplantation
defect of antigen-presentation to the B. Isotransplantation
immunocompetent cells was found C. Heterotransplantation
during investigation on the molecule D. Autotransplantation
level. Defect of what cells is the most E. Xenotransplantation
probable here?
A. Fibroblasts, Т-lymphocytes, В- 72. Continuous taking of some drugs
lymphocytes foregoing the pregnancy increase the
B. Т-lymphocytes, В-lymphocytes risk of giving birth to a child with genetic
C. NK-cells defects. What is this effect called?
D. Macrophages, monocytes A. Embryotoxic effect
E. 0-lymphocytes B. Blastomogenic effect
C. Mutagenic effect
69. Examination of cell culture got D. Fetotoxical effect
from a patient with lysosomal E. Teratogenic effect
pathology revealed accumulation of
great quantity of lipids in the 73. It was revealed that T-
lysosomes. What of the following lymphocytes were affected by HIV.
diseases is this disturbance typical Virus enzyme - reverse transcriptase
for? (RNA-dependent DNA polymerase) -
A. Tay-Sachs disease catalyzes the synthesis of:
B. Galactosemia A. Virus informational RNA on the
C. Wilson disease matrix of DNA
D. Phenylketonuria B. DNA on virus ribosomal RNA
E. Gout C. Viral DNA on DNA matrix
D. DNA on the matrix of virus mRNA

14
E. mRNA on the matrix of virus among the pupils who regularly did
protein physical exercises this index was only
20%. What adaptative mechanisms
74. A child was born with cleft determined such a low sickness rate of
palate. Examination revealed aorta pupils participating in the sports?
defects and reduced number of T- A. Specific adaptation
lymphocytes in blood. What B. Biochemical adaptation
immunodeficient syndrome is it? C. Physiological adaptation
A. Wiskott-Aldrich D. Cross adaptation
B. Chediak-Higashi E. Genetic adaptation
C. Louis-Bar
D. Swiss-type 78. After an immunoassay a child
E. DiGeorge was diagnosed with immunodeficiency
of humoral immunity. What is the
75. Examination of a child who reason for the primary
frequently suffers from infectious immunodeficiency development in the
diseases revealed that IgG child?
concentration in blood serum was 10 A. Toxic damage of B-lymphocytes
times less than normal, IgA and IgM B. Hereditary abnormality of immune
concentration was also significantly system
reduce D. Analysis showed also lack C. Pathometabolism in mother's
of B-lymphocytes and plasmocytes. organism
What disease are these symptoms D. Embryonal development
typical for? abnormalities
A. Swiss-type E. Immune responsiveness and
agammaglobulinemia resistance disorders
B. Louis-Bar syndrome
C. Dysimmunoglobulinemia 79. A pregnant woman was
D. Di George syndrome registered in an antenatal clinic and
E. Bruton's disease underwent complex examination for a
number of infections. Blood serum
76. A patient suffering from contained IgM to the rubella virus. What
infectious mononucleosis has been is this result indicative of?
taking glucocorticosteroids for two A. Of a chronic process
weeks. This resulted in remission but B. Of exacerbation of a chronic
the patient got exacerbation of chronic disease
tonsillitis. This complication is induced C. Of primary infection
by the following effect of D. The woman is healthy
glucocorticosteroids: E. Of recurring infection with rubella
A. Anti-shock virus
B. Antitoxic
C. Antiallergenic 80. Quite often the cause of
D. Immunosuppressive secondary immunodeficiency is an
E. Anti-inflammatory infection involvement, when the
causative agents propagate directly in
77. During influenza epidemic 40% the cells of immune system and destroy
of pupils who didn't go in for sports it. The following diseases are
were affected by the disease, and characterized by:

15
A. Q-febris, epidemic typhus B. Cytolysis, caused by lymphocytes
B. Dysentery, cholera C. Cytotoxic
C. Infectious mononucleosis, AIDS D. Reagin
D. Tuberculosis, mycobacteriosis E. Granulomatosis
E. Poliomyelitis, type A hepatitis
84. A woman complaining of
5. ALERGY coryza, phonastenia, eyelids redness
and lacrymation during spring period
came to the doctor. What type of
81. A 27-year-old woman has allergic reaction by Gell ano Coombs
dropped penicillin containing eye classification develops in this case?
drops. In few minutes there appeared A. Delayed type of hypersensitivity
feeling of itching, burning of the skin, B. Immunocomplex
lips and eyelids edema, whistling C. Stimulating
cough, decreasing of BP. What D. Cytotoxic
antibodies take part in the E. Anaphylactic
development of this allergic reaction?
A. IgA and IgM 85. A child with diphtheria 10 days
B. IgM and IgG after injection of antitoxic antidiphtherial
C. IgM and IgD serum has developed skin rash,
D. IgG and IgD accompanied by severe itch, rising
E. IgE and IgG temperature up to 38*0C and joints
pain. What is the cause of these
82. A patient with paroxysmal symptoms?
attacks of asphyxia, which appear A. Delayed type of hypersensitivity
after inhalation of different aromatic B. Anaphylactic reaction
substances has been made a C. Contact allergy
diagnosis of bronchial asthma. Ig E D. Atopia
rate is elevated. What type of reaction E. Serum sickness
is it the most typical for?
A. Anaphylactic 86. On the 8th day since the
B. Delayed type of hypersensitivity patient was inoculated with antitetanic
C. Autoimmune serum because of dirty wound of his
D. Cytotoxic foot he has developed rising
E. Immunocomplex 0
temperature up to 38 С, pains in the
joints, rash and itch. The blood tests
83. A patient who had been revealed leukopenia and
suffering for many years from thrombocytopenia. Allergic reaction of
bronchial asthma died from asphyxia. what type has developed in this case?
The histological examination of his A. Anaphylactic
lungs revealed the following: much B. Cytotoxic
mucus with eosinophiles contents in C. Delayed type of hypersensitivity
the lumen of bronchioles and small D. Stimulating
bronci, sclerosis of interalveolar septa, E. Immunocomplex
dilation of alveolar lumen. Which of
the mechanisms of allergy 87. A patient visited a dentist with
development is it? complaints of redness and edema of
A. Immunocomplex his mouth mucous membrane in a

16
month after dental prosthesis. The A. Cells cytotoxity
patient was diagnosed with allergic B. Reagin type cytotoxity
stomatitis. What type of allergic C. Antibody cytotoxity
reaction by Gell and Cumbs underlies D. Granuloma formation
this disease? E. Immunocomplex cytotoxity
A. Cytotoxic
B. Delayed type hypersensitivity 91. While enrolling a child to school
C. Anaphylactic Mantu's test was made to define
D. Stimulating whether revaccination was needed.
E. Immunocomplex The test result is negative. What does
this test result mean?
88. Donor skin transplantation was A. Absence of antitoxic immunity to
performed to a patient with extensive the tuberculosis
burns. On the 8-th day the graft B. Presence of antibodies for
became swollen and changed colour; tubercle bacillus
on the 11-th day graft rejection C. Absence of antibodies for
started. What cells take part in this tubercle bacillus
process? D. Presence of cell immunity to the
A. B-lymphocytes tuberculosis
B. Erythrocytes E. Absence of cell immunity to the
C. T-lymphocytes tuberculosis
D. Basophils
E. Eosinophils 92. A patient has got a spasm of
smooth muscles of bronchi. Activators
89. A 38-year-old patient died of what membrane cytoreceptors are
during intractable attack of bronchial phisiologically reasoned to stop an
asthma. Histologic examination attack?
revealed mucus accumulation in A. α-adrenoreceptors
bronchial lumen, a lot of fat cells B. N-cholinoreceptors
(labrocytes) in the wall of bronches, C. α and β-adrenoreceptors
many of them are in the state of D. M-cholinoreceptors
degranulation, there are also a lot of E. β-adrenoreceptors
eosinophils. What pathogenesis of
bronchial changes is it? 93. A woman has been applying a
A. Atopy new cosmetic preparation for a week
B. Cytotoxic, cytolytic action of that resulted in eye-lid inflammation
antibodies accompanied by hyperemia, infiltration
C. Granulomatosis and painfulness. What type of allergic
D. Immunocomplex mechanism reaction was developed?
E. Cellular cytolysis A. III
B. V
90. Tuberculine was injected C. II
intracutaneously to the child for D. I
tuberculin test. Marked hyperemia, E. ІV
tissue infiltration developed on the
place of injection in 24 hours. What 94. 48 hours after performing
mechanism caused these tuberculin test (Mantoux test) to a child
modifications? a 10 mm papule appeared on the spot

17
of tuberculin introduction. What 97. In course of histidine catabolism
hypersensitivity mechanism underlies a biogenic amin is formed that has
these changes? powerful vasodilatating effect. Name it:
A. Antibody-dependent cytotoxicity A. Noradrenalin
B. Granulomatosis B. Dioxyphenylalanine
C. Immune complex cytotoxicity C. Serotonin
D. Cellular cytotoxicity D. Dopamine
E. Anaphylaxis E. Histamine

95. An experimental animal was 98. A patient was stung by a bee.


first sensibilized whereupon an Examination revealed that his left hand
antigen dose was introduced was hot, pink, edematic, there was a
subcutaneously. This injection big red blister on the site of sting. What
resulted in the development of a is the leading mechanism of edema
fibrinous inflammation with alteration development?
of vessel walls, basal substance and A. Reduced vessel filling
fibrous structures of connective tissue B. Drop of osmotic pressure in tissue
in form of mucoid and fibrinoid C. Increased vessel permeability
swelling and necrosis. What D. Injury of vessels caused by the
immunological reaction took place? sting
A. Immediate hypersensitivity E. Drop of oncotic pressure in tissue
B. Reaction of transplantation
immunity 99. A female patient underwent liver
C. Delayed-type hypersensitivity transplantation. 1,5 month after it her
D. Normergic reaction condition became worse because of
E. Granulomatosis reaction of transplant rejection. What
factor of immune system plays the
96. A 30 year old woman has leading part in this reaction?
applied a lipstick with a fluorescent A. T-killers
substance for a long time. Then she B. B-lymphocytes
got a limited erythema and slight C. T-helpers
peeling on her lip border, later there D. Interleukin-1
appeared transversal striae and E. Natural killers
cracks. Special methods of
microscopic examination of the 100. A patient in a transplantation
affected area helped to reveal centre underwent heart transplantation.
sensibilized lymphocytes and The organ was taken from a donor who
macrophages in the connective tissue; died in a road accident. Foreign heart
cytolysis. What type of immunological can be rejected as a result of
hypersensitivity was developed? development of transplantation
A. IV type (cellular cytotoxicity) immunity. It is usually prevented by
B. Granulomatosis means of:
C. II type (antibody cytotoxicity) A. Immunosuppressors
D. I type (reaginic) B. X-ray therapy
E. III type (immune complex C. Chemotherapy
cytotoxicity) D. Enzymes
E. Ultrasound

18
101. A patient with skin mycosis positive skin graft. The graft was
has disorder of cellular immunity. The rejected in two weeks. Its microscopic
most typical characteristic of it is examination revealed circulatory
reduction of the following index: disturbance, edema and cellular
A. Plasmocytes infiltration with lymphocytes, neutrophils
B. Immunoglobulin E and macrophages predominance. What
C. Immunoglobulin G is the most likely pathology?
D. B-lymphocytes A. Interstitial inflammation
E. T-lymphocytes B. Delayed-type hypersensitivity
C. Graft immunity
102. A 50 year old man who was D. Immediate hypersensitivity
referred to the hospital for treatment of E. Granulomatous inflammation
cervical lymphadenitis underwent test
for induvidual sensitivity to penicillin. 105. During surgical manipulations a
30 seconds after he went hot all over, patient has been given novocaine
AP dropped down to 0 mm Hg that led injection for anesthesia. 10 minutes
to cardiac arrest. Resuscitation was later the patient developed paleness,
unsuccessful. Autopsy results: acute dyspnea, hypotension. What type of
venous plethora of internal organs; allergic reaction is it?
histological examination of skin (from A. Cellulotoxic immune reaction
the site of injection) revealed B. Cell-mediated immune reaction
degranulation of mast cells (tissue C. Stimulating immune reaction
basophils). Degranulation was also D. Anaphylactic immune reaction
revealed in myocardium and lungs. E. Aggregate immune reaction.
What type of hypersensitivity reaction
is it? 106. During hypersensitivity test a
A. Anaphylactic patient got subcutaneous injection of an
B. Complement-mediated cytotoxic antigen which caused reddening of
C. Delayed-type hypersensitivity skin, edema, pain as a result of
D. Immunocomplex-mediated histamine action. This biogenic amine
E. - is generated as a result of
transformation of the following histidine
103. Introduction of a big dose of amino acid:
histamine to an experimental animal A. Isomerization
caused abrupt drop of arterial B. Decarboxylation
pressure as a result of: C. Methylation
A. Constriction of resistance D. Deaminization
vessels E. Phosphorylation
B. Dilatation of resistance vessels
C. Increase of heart rate 107. A 10-year-old child had the
D. Decrease of heart rate mantoux tuberculin test administered.
E. Decrease of heart rate and force 48 hours later a papule up to 8 mm in
diameter appeared on the site of the
104. Examination of a pregnant injection. What type of hypersensitivity
woman having Rh-negative blood reaction developed after the tuberculin
revealed high level of antierythrocytic injection?
antibodies. For its reduction she was A. Type II hypersensitivity reaction
implanted with her husband's Rh- B. Atopic reaction

19
C. Arthus phenomenon D. Neurotonic
D. Type IV hypersensitivity reaction E. Metabolic
E. Seroreaction
111. During the expirement
performed on a rabbit the superior
6. Pathology of Macro- cervical node of sympathatic stem was
& Microcyrculation removed. On the side of the extraction
there is redness and rising temperature
of the head skin. What form of
108. Upper neck node of peripheral circulation disorder
sympathetic trunk was removed from developed in this case?
the rabbit on experiment. Reddening A. Neuroparalytic arterial hyperemia
and increased temperature of the skin B. Venous hyperemia
of head is observed. What form of C. Stasis
peripheral circulation of the blood D. Neurotonic arterial hyperemia
developed in the rabbit? E. Metabolic arterial hyperemia
A. Venous hyperemia
B. Stasis 112. Decreased blood supply to
C. Neurotonic arterial hyperemia the organs causes hypoxia that
D. Metabolic arterial hyperemia activates fibroblasts function. Volume of
E. Neuroparalytic arterial what elements is increased in this
hyperemia case?
A. Lymphatic vessels
109. Patient suffering from B. Parenchymatous elements of the
trombophlebitis of the deep veins organ
suddenly died. Autopsy has shown C. Nerve elements
freely lying red friable masses with D. Vessels of microcircular stream
dim crimped surface in the trunk and E. *Intercellular substance
bifurcation of the pulmonary artery.
What pathologic process was 113. During total (with water)
revealed by morbid anatomist? alimentary starvation the generalized
A. Fat embolism edemata have developed. Which of the
B. Thrombosis pathogenic factors is dominant in this
C. Embolism with foreign body case?
D. Tromboembolism A. Reduced oncolitic pressure of
E. Tissue embolism blood plasma
B. Reduced hydrostatic pressure of
110. A 42-year-old woman with interstitial fluid
trigeminal nerve neuralgia complaints C. Reduced osmotic pressure of
of redness of right part of her face and blood plasma
neck, feeling of heat rush and D. Increased oncolitic pressure of
increased dermal sensitiveness. What interstitial fluid
is the arterial hyperemia in this case E. Increased osmotic pressure of
according to pathophysiological interstitial fluid
mechanism?
A. Neuroparalytic 114. The concentration of albumins
B. Reactive in human blood sample is lower than
C. Working

20
normal. This leads to edema of C. Mixed
tissues. What blood function is D. Membranogenic
damaged? E. Lymphogenic
A. Maintaining the body
temperature 118. A 54-year-old man was
B. Maintaining the oncotic blood admitted to the hospital with complaints
pressure of pain in the right subcostal region,
C. Maintaining the blood vomiting with blood. Objectively:
sedimentation system enlarged liver, varicose veins in the
D. Maintaining the Ph level stomach and esophagus. Disfunction of
E. All answers are correct what vessel is likely to be?
A. Vena hepatica
115. A patient suffering from B. Vena cava superior
trombophlebitis of deep veins C. Vena cava inferior
suddenly died. The autopsy has D. Vena porta
shown freely lying red friable masses E. Aorta abdominalis
with dim crimped surface in the trunk
and bifurcation of the pulmonary 119. A patient with thrombophlebitis
artery. What pathologic process was of lower extremities had got chest
revealed by the morbid anatomist? pains, blood spitting, growing
A. Thrombosis respiratory failure that caused his
B. Embolism with foreign body death. Autopsy revealed multiple
C. Tromboembolism pulmonary infarctions. What is the most
D. Fat embolism probable reason of their development?
E. Tissue embolism A. Pulmonary artery thrombosis
B. Bronchial artery embolism
116. The pulmonalis embolism has C. Pulmonary venous thrombosis
suddenly developed in a 40-year-old D. Pulmonary artery embolism
patient with opened fracture of the hip. E. Bronchial artery thrombosis
Choose the possible kind of
embolism: 120. A hypertensive glucose solution
A. Thrombus-embolus was introduced to a patient. It will
B. Fat intensify water movement:
C. Tissue A. From the intercellular liquid to the
D. Air capillaries
E. Foreign body B. From the capillaries to the
intercellular liquid
117. Inflamation is characterised by C. From the cells to the intercellular
increased permeability of vessels of liquid
microcirculation stream, increase of D. From the intercellular liquid to the
their hydrodynamic blood pressure. cells
Increasing of the osmotic E. There will be no changes of water
concentration and dispersity of protein movement
structures is present in the
intercellular fluid. What kind of edema 121. While playing volleyball a
will appear in this case? sportsman made a jump and landed on
A. Colloid-osmotic the outside edge of his foot. He felt
B. Hydrodynamic acute pain in the talocrural joint, active

21
movements are limited, passive removed. Immediately after operation
movements are unlimited but painful. the temperature of ear skm was
A bit later there appeared a swelling in measured. !t was revealed that the
the area of external ankle, the skin temperature of the rabbit's ear skin on
became red and warm. What type of the side of denervation was by 1.5°C
peripheral circulation disturbance is higher than on the opposite intact side.
the case? What of the following is the most
A. Arterial hyperemia probable explanation of the above-
B. Venous hyperemia mentioned effects?
C. Stasis A. Arterial neurotopical hyperemia
D. Embolism B. Atrerial hyperemia induced by
E. Thrombosis metabolic factors
C. Physiological arterial hyperemia
122. A patient with nephrotic D. Arterial neuroparalytic hyperemia
syndrome has massive edemata of his E. Reactive arterial hyperemia
face and limbs. What is the leading
pathogenetic mechanism of edemata 125. Students who are taking
development? examinations often have dry mouth.
A. Increase of vascular The mechanism that causes this state
permeability is the realization of the following
B. Rise of hydrodynamic blood reflexes:
pressure A. Unconditioned peripheral
C. Increase of lymph outflow B. Unconditioned sympathetic
D. Lymphostasis C. Conditioned sympathetic
E. Drop of oncotic blood pressure D. Unconditioned parasympathetic
E. Conditioned parasympathetic
123. 2 hours after a skeletal
extension was performed to a 27 year 126. A patient with obliterating
old patient with multiple traumas atherosclerosis underwent
(closed injury of chest, closed fracture sympathectomy of femoral artery in the
of right thigh) his condition abruptly region of femoral trigone. What type of
became worse and the patient died arterial hyperemia was induced by the
from acute cardiopulmonary operation?
decompensation. Histological A. Functional
examination of pulmonary and B. Neurotonic
cerebral vessels stained with Sudan III C. Metabolic
revealed orange drops occluding the D. Neuroparalytic
vessel lumen. What complication of E. Reactive
polytrauma was developed?
A. Fat embolism 127. A patient ill with
B. Thromboembolism thrombophlebitis of his lower limbs had
C. Gaseous embolism chest pain, blood spitting, progressing
D. Microbal embolism respiratory insufficiency that led to his
E. Air embolism death. Autopsy diagnosed multiple lung
infarctions. What is the most probable
124. A rabbit's nerve that cause of their development?
innervates the right ear was cut and A. Thromboembolism of bronchial
its right superior cervical ganglion was arteries

22
B. Thrombosis of pulmonary artery 7. INFLAMATION
branches
C. Thrombosis of bronchial arteries
131. Aspirin has antiinflammatory
D. Thrombosis of pulmonary veins
effect due to inhibition of the
E. Thromboembolism of pulmonary
cyclooxygenase activity. Level of what
artery branches
biological active acids will decrease?
A. Biogenic amines
128. A student failed to answer all
B. Catecholamines
the questions of examination paper
C. Prostaglandins
correctly. As a result he blushed, felt
D. Leucotriens
hot and lost confidence. What type of
E. Iodinethyronyns
arterial hyperemia has developed in
this case?
132. Tuberculine was injected
A. Postishemic hyperemia
intraperitoneally to the animal
B. Neurotonic hyperemia
sensibilized with tuberculine. Venous
C. Pathologic hyperemia
hyperemia and peritonial edema were
D. Metabolic hyperemia
detected on the laparotomy in 24 hours.
E. Neuroparalytic hyperemia
Increased amount of lymphocytes and
monocytes were in the smear-print from
129. Autopsy of a 73-year-old man
the peritonium. What pathological
who had been suffering from the
process is in the animal?
coronary heart disease along with
A. Suppurative inflammation
cardiac insufficiency for a long time
B. Fibrinous inflammation
revealed: nutmeg liver, brown
C. Serous inflammation
induration of lungs, cyanotic induration
D. Aseptic inflammation
of kidneys and spleen. What kind of
E. Allergic inflammation
circulation disorder was the cause of
such effects?
133. Necrosis focus appeared in the
A. General acute venous
area of hyperemia and skin edema in
congestion
few hours after burn. What mechanism
B. Acute anaemia
strengthens destructive events in the
C. General chronic venous
inflammation area?
congestion
A. Proliferation of fibroblasts
D. Chronic anaemia
B. Secondary alteration
E. Arterial hyperaemia
C. Primary alteration
D. Emigration of lymphocytes
130. A 45-year-old woman has
E. Diapedesis of erythrocytes
breast cancer. Her left arm has
symptoms of lymphatic system
134. On simulation of inflammation
insufficiency - limb edema, lymph
of the lower extremity the animal
node enlargement. What form of
exrerienced raise of the temperature,
lymphatic circulation insufficiency is it?
increase of amount of antibodies and
A. Combined insufficiency
leucocytes in the blood. What
B. Dynamic insufficiency
substances caused this general
C. Resorption insufficiency
reaction of the organism on
D. Mechanic insufficiency
inflammation?
E. -
A. Glucocorticoid

23
B. Leucotriens A. Autoimmune inflammation
C. Interleukin B. Vessels thrombosis
D. Mineralcorticoid C. Secondary infection
E. Somatomedins D. Ischemia of myocardium
E. Resorption of enzymes from
135. Necrosis focus was observed necrotized area of myocardium
in the area of hyperemia and skin
edema in a few hours after burn. What 139. A patient has undergone an
mechanism strengthens destructive amputation of lower extremity. Some
effects in the inflammation area? time later painful nodules appeared in a
A. Proliferation of fibroblasts stump. Amputatious neuromas were
B. Diapedesis of erythrocytes found out at the microscopic
C. Primary alteration examination. To what pathological
D. Secondary alteration processes do those formations relate?
E. Emigration of lymphocytes A. Regeneration
B. Dystrophy
136. Inflamation is characterised by C. Hyperemia
increasing penetration of vessels of D. Inflammation
microcirculation stream, increasing of E. Metaplasia
their fluid dynamic blood pressure.
Increasing of the osmotic 140. A 16-year-old boy was
concentration and dispersity of protein performed an appendectomy. He has
structures can be found in the been hospitalized for right lower
intercellular fluid. What kind of edema quadrant abdominal pain within 18
are to be observed in this case? hours. The surgical specimen is
A. Membranogenic edematous and erythematous.
B. Lymphogenic Infiltration by what of the following cells
C. Mixed is the most typical for the process
D. Colloid-osmotic occuring here?
E. Hydrodynamic A. Basophils
B. Monocytes
137. At the aboratory experiment C. Limphocytes
the leukocyte culture was mixed with D. Eosinophils
staphylococci. Neutrophile leukocytes E. Neutrophils
engulfed and digested bacterial cells.
This processes are termed: 141. A 17-year-old boy fell seriously
A. Facilitated diffusion ill, body temperature rose up to 38,50С,
B. Diffusion there is cough, rhinitis, lacrimation,
C. Osmosis nasal discharges. What kind of
D. Phagocytosis inflammation is it?
E. Pinocytosis A. Fibrinous inflammation
B. Hemorrhagic inflammation
138. A patient in three weeks after C. Suppurative inflammation
acute myocardial infarction has pain in D. Catarrhal inflammation
the heart and joints and pneumonia. E. Serous inflammation
What is the main mechanism of
development of post-infarction 142. A 5-year-old child who often
Dressler’s syndrome? fells ill with respiratory diseases has

24
eczematous appearances after 146. In course of an experiment a
consumption of some food products, big number of column cells of red bone
tendency to prolonged course of marrow was in some way destructe d.
inflammatory processes. What kind of Regeneration of which cell populations
diathesis can be suspected in this in the loose connective tissue will be
case? inhibited?
A. Exudative-catharral A. Of pericytes
B. Hemmorhagic B. Of fibroblasts
C. Lymphohypoplastic C. Of lipocytes
D. Asthenic D. Of pigment cells
E. Arthritism E. Of macrophags

143. A patient with infectious 147. A 17 year old boy fell seriously
mononucleosis had been taking ill, the body temperature rose up to
glucocorticoids for two weeks. He was 38,5oC there appeared cough, rhinitis,
brought into remission, but he fell ill lacrimation, nasal discharges. What
with acute attack of chronic tonsillitis. inflammation is it?
What action of glucocorticoids caused A. Serous
this complication? B. Purulent
A. Immunosuppressive C. Fibrinous
B. Antiinflammatory D. Hemorrhagic
C. Antishock E. Catarrhal
D. Antiallergic
E. Antitoxic 148. Tuberculine was injected
intraperitoneally to the animal
144. Inflammation of a patient's eye sensibilized with tuberculine. Venous
was accompanied by accumulation of hyperemia and peritonial edema were
turbid liquid with high protein at the detected on the laparotomy in 24 hours.
bottom of anterior chamber that was Increased amount of lymphocytes and
called hypopyon. What process monocytes were in the smear-print from
underlies the changes under the peritonium. What pathological
observation? process is in the animal?
A. Secondary alteration A. Serous inflammation
B. Disturbance of microcirculation B. Suppurative inflammation
C. Primary alteration C. Allergic inflammation
D. Proliferation D. Fibrinous inflammation
E. - E. Aseptic inflammation

145. Utilization of arachidonic acid 149. Inflammatory processes cause


via cyclooxigenase pathway results in synthesis of protein of acute phase in
formation of some bioactive an organism. What substances
substances. Name them: stumulate their synthesis?
A. Somatomedins A. Interleukin-1
B. Biogenic amins B. Angiotensin
C. Thyroxine C. Interferons
D. Prostaglandins D. Immunoglobulins
E. Insulin-like growth factors E. Biogenic amins

25
150. A 4 year old child complained weight loss, hoarse voice, dyspnea, dry
of pain during deglutition, cough. Laryngoscopy revealed a
indisposition. Objectively: palatine tumour in the pharynx that invaded
arches and tonsils are moderately vocal cords and epiglottis. What is the
edematic and hyperemic, there are most probable cause of tumour
greyish-white films up to 1 mm thick development?
closely adhering to the subjacent A. Nitrosamines
tissues. What pathological process B. Polycyclic aromatic carbohydrates
are these changes typical for? C. Retroviruses
A. Dystrophy D. Ionizing radiation
B. Inflammation E. Aromatic amines and amides
C. Metaplasia
D. Organization 154. An alcoholic woman has born a
E. Necrosis girl with mental and physical
developmental lag. Doctors diagnosed
151. Blood plasma of a healthy the girl with fetal alcohol syndromу.
man contains several dozens of What effect is the cause of the girl's
proteins. During an illness new state?
proteins can originate, namely the A. Malignization
protein of "acute phase". Select such B. Carcinogenic
protein from the listed below: C. Mechanic
A. C-reactive protein D. Teratogenic
B. Prothrombin E. Mutagenic
C. G immunoglobulin
D. Fibrinogen 155. A patient who has been
E. A immunoglobulin abusing tobacco smoking for a long
time has got cough accompanied by
8. Pathophysiology of excretion of viscous mucus; weakness
after minor physical stress, pale skin.
Tumor Growth The patient has also lost 12,0 kg of
body weight. Endoscopic examination
152. A 62 y.o. woman complains of of biosy material his illness was
frequent pains in the area of her chest diagnosed as squamous cell
and backbone, rib fractures. A doctor carcinoma. Name a pathological
assumed myelomatosis process that preceded formation of the
(plasmocytoma). What of the following tumour:
laboratory characteristics will be of the A. Metaplasia
greatest diagnostical importance? B. Hyperplasia
A. Proteinuria C. Hypoplasia
B. Hypoproteinemia D. Sclerosis
C. Hypoglobulinemia E. Necrosis
D. Hyperalbuminemia
E. Paraproteinemia 9. Cell ingury
153. A 62 year old patient who
156. Due to action of electric current
previously worked as stoker was
on the exitable cell there appeared
admitted to a hospital with complaints
depolarization of it's membrane.
about general weakness, abrupt

26
Movement of what ions through the D. Transport of fatty acids to the
membrane caused depolarisation? mitochondria
A. НСО3- E. Synthesis of steroid hormones
B. Na+
C. Са2+ 161. Low level of albumins and
D. Сl- fibrinogen was detected in the patient's
E. К+ blood. Decreased activity of what
organelle of the liver hepatocytes can
157. During metabolic process, cause it?
active forms of the oxygen including A. Mitochondrions
superoxide anion radical are formed in B. Golgi complex
the human body. With help of what C. Lysosomes
enzyme is this anion activated? D. Granular endoplasmatic net
A. Peroxidase E. Agranular endoplasmatic net
B. Glutathioneperoxidase
C. Superoxide dismutase 162. The penetration of the irritable
D. Catalase cell membrane for potassium ions has
E. Glutathionereductase been increased during an experiment.
What changes of membrane electric
158. Highly injured person status can occur?
gradually died. Please choose the A. Local response
indicator of biological death: B. Action potential
A. Disarray of chemical processes C. No changes
B. Loss of consciousness D. Depolarization
C. Absence of palpitation E. Hyperpolarization
D. Autolysis and decay in the cells
E. Absence of movements 163. The preventive radioprotector
was given to a worker of a nuclear
159. Due to activation of ion power station. What mechanism from
channels of external membrane of the below mentioned is considered to
excitable cell it's rest potential has be the main mechanism of
significantly increased. What channels radioprotection?
were activated? A. Increasing of respiration
A. Fast calcium channels B. Inhibition of free radicals
B. Potassium channels formation
C. Natrium and calcium channels C. Activation of oxidation reactions
D. Slow calcium channels D. Prevention of tissue’s hypoxia
E. Natrium channels E. Increasing of tissue blood supply

160. Carnitine including drug was 164. The formation of a secondary


recomended to the sportsman for mediator is obligatory in membrane-
improving results. What process is intracellular mechanism of hormone
activated most of all with help of action. Point out the substance that is
carnitine? unable to be a secondary mediator:
A. Synthesis of ketone bodies A. Diacylglycerol
B. Synthesis of lipids B. CAMP
C. Tissue respiration C. Inositol-3,4,5-triphosphate
D. Ca2+

27
E. Glycerol C. Wilson disease
D. Tay-Sachs disease
165. During the experiment on the E. Galactosemia
influence of chemical substances in
the muscles the reaction of Ca2+-pump 169. Normal, actively dividing cells
is weakened. Which phenomenum will of human red bone marrow are
be observed? analyzed. What number of cells'
A. Decreased velocity of the AP chromosomes is typical for G1 period?
distribution A. 46
B. Activation of the sodium- B. 48
potassium pump C. 23
C. Prolonged relaxation D. 45
D. Decreased AP E. 47
E. Prolonged duration of the AP
170. An isolated muscle fiber is
166. A patient who was previously under examination. It was established
ill with mastectomy as a result of that the threshold of stimulation force
breast cancer was prescribed became significantly lower. What is the
radiation therapy. What vitamin cause of this phenomenon?
preparation has marked A. Inactivation of sodium channels of
radioprotective action caused by membrane
antioxidant activity? B. Inactivation of potassium
A. Thiamine chloride channels of membrane
B. Ergocalciferol C. Block of energy production in the
C. Tocopherol acetate cell
D. Riboflavin D. Activation of potassium channels
E. Folic acid of membrane
E. Activation of sodium channels of
167. ATP synthesis is totaly membrane
blocked in a cell. How will the value of
membrane rest potential change? 171. Labelled amino acids alanine
A. It will be slightly increased and tryptophane were injected to a
B. First it will increase, then mouse in order to study localization of
decrease protein synthesis in its cells. The
C. First it will decrease, then labelled amino acids will be
increase accumulated near the following
D. It will be considerably increased organellas:
E. It will disappear A. Ribosomes
B. Smooth endoplasmic reticulum
168. Examination of cell culture got C. Cell centre
from a patient with lysosomal D. Golgi apparatus
pathology revealed accumulation of E. Lysosomes
great quantity of lipids in the
lysosomes. What of the following 172. Study of conversion of a food
diseases is this disturbance typical colouring agent revealed that
for? neutralization of this xenobiotic takes
A. Gout place only in one phase - microsomal
B. Phenylketonuria

28
oxydation. Name a component of this E. With domination of proteins and
phase: fat
A. Cytochrome C
B. Cytochrome B 176. A 4 y.o. child with signs of
C. Cytochrome A durative proteinic starvation was
D. Cytochrome Р-450 admitted to the hospital. The signs
E. Cytochrome oxidase were as follows: growth inhibition,
anemia, edemata, mental deficiency.
173. As a result of activation of the Choose a cause of edemata
ion channels of the external development:
membrane the rest potential of an A. Reduced synthesis of hemoglobin
excitable cell has greatly increased. B. Reduced synthesis of albumins
What channels were activated? C. Reduced synthesis of lipoproteins
A. Sodium and calcium channels D. Reduced synthesis of globulins
B. Fast calcium channels E. Reduced synthesis of
C. Potassium channels glycoproteins
D. Sodium channels
E. Slow calcium channels 177. As a result of long-term
starvation the glomerular filtration of a
174. There are several groups of man was accelerated by 20%. The
molecular mechanisms playing most probable cause of filtration
important part in pathogenesis of changes under such conditions is:
insult to cells which contributes to the A. Rise of systemic arterial pressure
pathology development. What B. Growth of filtration coefficient
processes are stimulated by proteinic C. Increase of renal plasma flow
damage mechanisms? D. Increased permeability of renal
A. Acidosis filter
B. Phospholipase activation E. Fall of oncotic pressure of blood
C. Osmotic membrane distension plasma
D. Lipid peroxidation
E. Enzyme inhibition 178. Power inputs of a boy
increased from 500 to 2000 kj pro hour.
10. Starvation & What can be the cause of it?
A. Transition from sleep to
Pathology of Metabolism wakefulness
B. Mental activity
175. Respiratory coefficient was C. Raise of outer temperatute
studied in the patient who strictly kept D. Physical exercise
certain diet for 10 days. It was E. Food intake
determined that it is 1. What diet does
the patient follow? 179. Long-term starvation cure of a
A. With domination of fat and patient resulted in diminished ratio of
carbohydrates albumines and globulines in plasma.
B. With domination of proteins and What of the following will be result of
carbohydrates these changes?
C. With domination of A. Increase of hematocrit
carbohydrates B. Increase of ESR
D. Mixed C. Decrease of hematocrit

29
D. Hypercoagulation B. Decrease in the oncotic pressure
E. Decrease of ESR of blood plasma
C. Increase in the systemic arterial
180. Dietary intake of a 30 year old pressure
nursing woman contains 1000 mg of D. Increase of the filtartion quotient
calcium, 1300 mg of phosphorus and E. Increase of the renal blood flow
20 mg of iron per day. It is necessary
to change content of these mineral 11. Disorders of
substances in the following way:
A. To increase calcium content Carbohydrate Metabolism
B. To reduce fluorine content
C. To reduce iron content 184. On the empty stomach in the
D. To increase iron content patients blood glucose level was 5,65
E. To increase phosphorus content mmol/L, in an hour after usage of sugar
it was 8,55 mmol/L, in a 2 hours - 4,95
181. When measuring power inputs mmol/L. Such indicators are typical for:
of a man by the method of indirect A. Patient with non-insulin
calorimetry the following results were dependent diabetes mellitus
obtained: 1000 ml oxygen B. Patient with insulin-dependent
consumption and 800 ml carbon diabetes mellitus
dioxide liberation per minute. The man C. Patient with tireotoxicosis
under examination has the following D. Patient with hidden diabetes
respiratory coefficient: mellitus
A. 1,0 E. Healthy person
B. 0,9
C. 0,8 185. Patient with diabetes didn't get
D. 1,25 insulin injection in time that caused
E. 0,84 hyperglycemic coma (glucose in the
blood 50mmol/L). What mechanism is
182. During starvation muscle prevalent in the development of the
proteins break up into free amino coma?
acids. These compounds will be the A. Hypokaliemia
most probably involved into the B. Hypoxia
following process: C. Hyperosmia
A. Synthesis of higher fatty acids D. Hyponatremia
B. Glycogenolysis E. Acidosis
C. Decarboxylation
D. Gluconeogenesis in liver 186. The B cells of endocrine portion
E. Gluconeogenesis in muscles of pancreas are selectively damaged by
alloxan poisoning. How will it be
183. As a result of continuous reflected in blood plasma?
starvation the glomerular filtration rate A. The content of globulins
has increased by 20%. The most decreases
probable cause of the glomerular B. The content of albumins
filtration alteration under the decreases
mentioned conditions is: C. The content of fibrinogen
A. Increase in the permeability of decrease
the renal filter D. The level of sugar decreases

30
E. The content of sugar increases A. Ketone bodies
B. Monosaccharides
187. A patient with the symptoms C. Cholesterol esters
of acute alcoholic poisoning was D. High fatty acids
brought to the hospital. What E. Amino acids
carbohydrates metabolism changes
are typical for this condition? 191. Diabetes mellitus causes
A. The anaerobic glucose ketosis as a result of activated
metabolism predominates in muscles oxidation of fatty acids. What disorders
B. The gluconeogenesis is of acid-base equilibrium may be caused
increased in liver by excessive accumulation of ketone
C. The breakage of glycogen is bodies in blood?
increased in liver A. Respiratory acidosis
D. The gluconeogenesis velocity in B. Metabolic alcalosis
liver is decreased C. Any changes woun't happen
E. The anaerobic breakage of D. Metabolic acidosis
glucose is increased in muscles E. Respiratory alcalosis

188. Patient with diabetes mellitus 192. A patient was delivered to the
experienced loss of consciousness hospital by an emergency team.
and convulsions after injection of Objectively: grave condition,
insulin. What is the result of unconscious, adynamy. Cutaneous
biochemical blood analysis for surfaces are dry, eyes are sunken, face
concentration of the sugar? is cyanoti c. There is tachycardia and
A. 10,0 mmol/L smell of acetone from the mouth.
B. 3,3 mmol/L Analysis results: blood glucose - 20,1
C. 5,5 mmol/L micromole/l (standard is 3,3-5,5
D. 1,5 mmol/L micromole/l), urine glucose - 3,5%
E. 8,0 mmol/L (standard is - 0). What is the most
probable diagnosis?
189. The gluconeogenesis is A. Acute alcoholic intoxication
activated in the liver after intensive B. Anaphylactic shock
physical trainings .What substance is C. Hyperglycemic coma
utilized in gluconeogenesis first of all D. Acute heart failure
in this case: E. Hypoglycemic coma
A. Glucose
B. Glutamate 193. A child is languid, apathetic.
C. Alanine Liver is enlarged and liver biopsy
D. Lactate revealed a significant excess of
E. Pyruvate glycogene. Glucose concentration in
the blood stream is below normal. What
190. The patient with diabetes is the cause of low glucose
mellitus has been delivered in hospital concentration?
in the state of unconsciousness. A. Low (absent) activity of
Arterial pressure is low. The patient hexokinase
has acidosis. Point substances, which B. High activity of glycogen
accumulation in the blood results in synthetase
these manifestations:

31
C. Deficit of a gene that is prednisolone for a long time.
responsible for synthesis of glucose 1- Examination revealed high rate of
phosphaturidine transferase sugar in his blood. This complication is
D. Low (absent) activity of caused by the drug influence upon the
glycogene phosphorylase in liver following link of carbohydrate
E. Low (absent) activity of glucose metabolism:
6-phosphatase A. Intensification of glucose
absorption in the bowels
194. A patient is ill with diabetes B. Inhibition of glycogen synthesis
mellitus that is accompanied by C. Gluconeogenesis activation
hyperglycemia of over 7,2 millimole/l D. Glycogenogenesis activation
on an empty stomach. The level of E. Activation of insulin
what blood plasma protein allows to decomposition
estimate the glycemia rate
retrospectively (4-8 weeks before 198. A 12-year-old teenager has
examination)? significantly put off weight within 3
A. C-reactive protein months; glucose concentration rose up
B. Ceruloplasmin to 50 millimole/l. He fell into a coma.
C. Fibrinogen What is the main mechanism of its
D. Albumin development?
E. Glycated hemoglobin A. Hypoglycemic
B. Ketonemic
195. An experimental animal has C. Hypoxic
been given excessive amount of D. Lactacidemic
carbon-labeled glucose for a week. E. Hyperosmolar
What compound can the label be
found in? 12. Disorders of Lipid
A. Choline
B. Methionine Metabolism
C. Palmitic acid
D. Vitamin A 199. A 65-year-old man with
E. Arachidonic acid signs of profuse obesity and risk of fatty
liver dystrophy was recommended
196. Characteristic sign of high-lipotropic diet. The contents of
glycogenosis is muscle pain during what substance in his diet is the most
physical work. Blood examination effective in this case?
reveals usually hypoglycemia. This
A. Glycin
pathology is caused by congenital
deficiency of the following enzyme: B. Methionine
A. Glycogen phosphorylase C. Glucose
B. Lysosomal glycosidase D. Cholesterol
C. Gamma amylase E. Vitamin C
D. Glucose 6-phosphate
dehydrogenase
E. Alpha amylase 200. A 70 year old man is ill with
vascular atherosclerosis of lower
197. A patient ill with extremities and coronary heart disease.
neurodermatitis has been taking Examination revealed disturbance of

32
lipidic blood composition. The main 203. Periodic renal colics attackes
factor of atherosclerosis pathogenesis are observed in the woman with
is the excess of the following primery hyperparathyroidizm.
lipoproteins: Ultrasonic examination revealed small
A. Low-density lipoproteins stones in the kidneys. What is the
B. Intermediate density lipoproteins cause of the formation of the stones?
A. Hypercholesterinemia
C. Cholesterol B. Hyperkalemia
D. High-density lipoproteins C. Hypercalcemia
E. Chylomicrons D. Hyperphosphatemia
E. Hyperuricemia
201. Increased amount of free fat
204. A 56-year-old patient with
acids is observed in the blood of the
cardiac failure notices edematous feet
patients with diabetes mellitus. It can
and shins, the skin in the place of
be caused by:
edema is pale and cold. What is the
A. Activation of the ketone bodies leading pathogenesis of this patient's
utilization edema?
B. Activation of the synthesis of the A. Increased hydrostatic pressure in
apolipoproteins venulaes
C. Increased activity of B. Reduced oncotic pressure in
triglyceridelipase adipocytes capillaries
D. Storage of palmitatoil-CoA C. Lymph outflow impairment
D. Increased capillary penetration
E. Decreased activity of
E. Positive water balance
phosphatidylcholine-cholesterol-
acyltransferase blood plasma
205. During total (with water)
alimentary starvation the generalized
13. Disorders of edemata have developed. Which of the
pathogenic factors is dominant in this
Water-Electrolite Balance case?
A. Reduced oncolitic pressure of
202. Inflamation is characterised by blood plasma
increasing penetration of vessels of B. Reduced hydrostatic pressure of
microcirculation stream, increasing of interstitial fluid
their fluid dynamic blood pressure. C. Reduced osmotic pressure of
Increasing of the osmotic blood plasma
concentration and dispersity of protein D. Increased oncolitic pressure of
structures present in the intercellular interstitial fluid
fluid. What kind of edema will appear E. Increased osmotic pressure of
in this case? interstitial fluid
A. Hydrodynamic
B. Lymphogenic 206. Chronic glomerulonephritis was
C. Membranogenic diagnosed in a 34-year-old patient 3
D. Colloid-osmotic years ago. Edema has developed for
E. Mixed the last 6 months. What caused it?
A. Hyperosmolarity of plasma
B. Hyperproduction of vasopressin

33
C. Proteinuria C. Decrease of oncotic pressure of
D. Disorder the livers protein blood plasma
formation function D. Increase of systemic blood
E. Hyperaldosteronism pressure
E. Increase of renal plasma stream
207. A 50-year-old patient
complains of thirst, drinking of a lot of 211. The concentration of albumins
water, marked polyuria. Blood glucose in human blood sample is lower than
is 4,8 mmol/L, urine glucose and normal. This leads to edema of tissues.
acetone bodies are absent, urine is What blood function is damaged?
colorless, specific gravity is 1,002- A. Maintaining the body temperature
1,004. What is the cause of polyuria? B. Maintaining the oncotic blood
A. Hypothyroidism pressure
B. Thyrotoxicosis C. Maintaining the blood
C. Aldosteronism sedimentation system
D. Vasopressin insufficiency D. Maintaining the Ph level
E. Insulin insufficiency E. All answers are correct

208. A man after 1,5 litre blood loss 212. A person felt thirsty after
has suddenly reduced diuresis. The staying under the conditions of hot
increased secretion of what hormone weather for a long time. Signals of what
caused such diuresis alteration? receptors caused it first of all?
A. Vasopressin A. Osmoreceptors of hypothalamus
B. Corticotropin B. Baroreceptors of aortic arch
C. Cortisol C. Osmoreceptors of liver
D. Parathormone D. Sodium receptors of
E. Natriuretic hypothalamus
E. Glucoreceptors of hypothalamus
209. A 2-year-old child experienced
convulsions because of lowering 213. A patient who suffers from
calcium ions concentration in the heart failure has enlarged liver,
blood plasma. Function of what edemata of lower extremities, ascites.
structure is decreased? What is the leading mechanism in the
A. Pineal gland development of this edema?
B. Thymus A. Lymphogenous
C. Hypophysis B. Membranogenic
D. Adrenal cortex C. Hydrodynamic
E. Parathyroid glands D. Colloid osmotic
E. –
210. Glomerular filtration rate
(GFR) increased by 20% due to 214. Osmotic pressure of a man's
prolonged starvation of the person. blood plasma is 350 mosmole/l
The most evident cause of filtration (standard pressure is 300 mosmole/l).
changes under this conditions is: First of all it will result in high secretion
A. Increase of filtration coefficient of the following hormone:
B. Increase of penetration of the A. Aldosteron
renal filter B. Cortisol
C. Adrenocorticotropin

34
D. Vasopressin pathogenetic mechanism of edemata
E. Natriuretic development?
A. Increase of vascular permeability
215. A patient has a decreased B. Rise of hydrodynamic blood
vasopressin synthesis that causes pressure
polyuria and as a result of it evident C. Increase of lymph outflow
organism dehydratation. What is the D. Lymphostasis
mechanism of polyuria development? E. Drop of oncotic blood pressure
A. Acceleration of glomerular
filtration 219. A concentrated solution of
B. Reduced tubular reabsorption of sodium chloride was intravenously
protein injected to an animal. This caused
C. Reduced tubular reabsorption of decreased reabsorption of sodium ions
Na ions in the renal tubules. It is the result of
D. Reduced glucose reabsorption the following changes of hormonal
E. Reduced tubular reabsorption of secretion:
water A. Aldosterone increase
B. Vasopressin increase
216. A patient ill with enteritis C. Aldosterone reduction
accompanied by massive diarrhea has D. Vasopressin reduction
low water rate in the extracellular E. Reduction of atrial natriuretic
space, high water rate inside the cells factor
and low blood osmolarity. What is
such disturbance of water-electrolytic 220.Decreased blood supply to the
metabolism called? organs causes hypoxia that activates
A. Hyperosmolar hypohydration fibroblasts function. Volume of what
B. Hypo-osmolar hyperhydration elements is increased in this case?
C. Hypo-osmolar hypohydration A. Nerve elements
D. Hyperosmolar hyperhydration B. Parenchymatous elements of the
E. Osmolar hypohydration organ
C. Lymphatic vessels
217. As a result of long-term D. Intercellular substance
starvation the glomerular filtration of a E. Vessels of microcircular stream
man was accelerated by 20%. The
most probable cause of filtration 221. A hypertensive glucose solution
changes under such conditions is: was introduced to a patient. It will
A. Rise of systemic arterial intensify water movement:
pressure A. From the intercellular liquid to the
B. Growth of filtration coefficient cells
C. Increase of renal plasma flow B. From the cells to the intercellular
D. Increased permeability of renal liquid
filter C. From the intercellular liquid to the
E. Fall of oncotic pressure of blood capillaries
plasma D. From the capillaries to the
intercellular liquid
218. A patient with nephrotic E. There will be no changes of water
syndrome has massive edemata of his movement
face and limbs. What is the leading

35
222. A patient was stung by a bee. 225. A patient was admitted to the
Examination revealed that his left infectious department. His symptoms:
hand was hot, pink, edematic, there dry skin, decreased skin turgor, rice-
was a big red blister on the site of water stool. The patient was diagnosed
sting. What is the leading mechanism with choler A. What disorder of water-
of edema development? electrolytic balance is most often
A. Reduced vessel filling observed in this disease?
B. Drop of osmotic pressure in A. Hyperosmotic hyperhydration
tissue B. Hyperosmotic hypohydration
C. Increased vessel permeability C. Hypoosmotic hypohydration
D. Injury of vessels caused by the D. Hypoosmotic hyperhydration
sting E. Isoosmotic hypohydration
E. Drop of oncotic pressure in
tissue 226. A 56 year old patient suffering
from cardiac insufficiency has edema of
223. A man weighs 80 kg, after feet and shins, edematous skin is pale
long physical activity his circulating and cold. What is the leding
blood volume is reduced down to 5,4 l, mechanism of edema pathogenesis?
hematocrit makes up 50%, whole A. Increase of capillary permeability
blood protein is 80 g/l. These blood B. Rise of hydrostatic pressure in
characteristics are determined first of venules
all by: C. Drop of oncotic pessure in
A. Water loss with sweat capillaries
B. Increased diuresis D. Disorder of lymph outflow
C. Increased number of E. Positive water balance
erythrocytes
D. Increased circulating blood 227. A patient is 44 years old.
volume Laboratory examination of his blood
E. Increased protein concentration revealed that content of proteins in
in plasm plasma was 40 g/l. What influence will
be exerted on the transcapillary water
224. A patient is 44 years old. metabolism?
Laboratory examination of his blood A. Metabolism will stay unchanged
revealed that content of proteins in B. Filtration will be increased,
plasma was 40 g/l. What influence will reabsorption - decreased
be exerted on the transcapillary water C. Both filtration and reabsorption
exchange? will be increased
A. Filtration will be decreased, D. Filtration will be decreased,
reabsorption - increased reabsorption - increased
B. Both filtration and reabsorption E. Both filtration and reabsorption
will be decreased will be decreased
C. Both filtration and reabsorption
will be increased 228. A patient with enteritis
D. Exchange will stay unchanged accompanied by massive diarrhea has
E. Filtration will be increased, low water rate in the extracellular
reabsorption - decreased space, high water rate inside the cells
and low blood osmolarity. What is such

36
disturbance of water-electrolytic 232. Diabetes mellitus causes
metabolism called? ketosis as a result of activated
A. Hypo-osmolar hyperhydration oxidation of fatty acids. What disorders
B. Hypo-osmolar hypohydration of acid-base equilibrium may be caused
C. Hyperosmolar hyperhydration by excessive accumulation of ketone
D. Hyperosmolar hypohydration bodies in blood?
E. Osmolar hypohydration A. Respiratory acidosis
B. Metabolic alcalosis
14. Disorders of Acid- C. Any changes woun't happen
D. Metabolic acidosis
Base Balance E. Respiratory alcalosis

229. Index of pH of the blood 233. A pregnant woman had been


changed and became 7,3 in the having toxicosis with severe repeated
patient with diabetus mellitus. vomiting for 24 hours. In the end of the
Detecting of the components of what day there appeared tetanic convulsions
buffer system is used while and fluid loss. What shift of acid-base
diagnosing disorder of the acid-base state caused these changes?
equilibrium? A. Gaseous acidosis
A. Protein B. Metabolic acidosis
B. Oxyhemoglobin C. Excretory alkalosis
C. Phosphate D. Gaseous alkalosis
D. Hemoglobin E. Excretory acidosis
E. Bicarbonate
234. A group of mountain climbers
230. Disorder of the airways went through the blood analysis at the
passage in small and middle bronchi height of 3000 m. It revealed decrease
was revealed in the patient. What of HCO3 to 15 micromole/l (standard is
disorder of the acid-base equilibrium 22-26 mmole/l). What is the mechanism
can be detected in the blood? of HCO3 decrease?
A. Respiratory alkalosis A. Hyperventilation
B. Metabolic alkalosis B. Decrease of bicarbonate
C. Metabolic acidosis reabsorption in kidneys
D. Respiratory acidosis C. Hypoventilation
E. – D. Decrease of ammoniogenesis
E. Intensification of acidogenesis
231. A child, suffering from
pylorostenosis accompanied by 235. As a result of exhausting
frequent vomiting, developed signs of muscular work a worker has largely
dehydration. What form of acid-base reduced buffer capacity of blood. What
disbalance can develop in this case? acidic substance that came to blood
A. Gaseous alkalosis caused this phenomenon?
B. Gaseous acidosis A. Pyruvate
C. Nongaseous acidosis B. 1.3-bisphosphoglycerate
D. Nongaseous alkalosis C. Lactate
E. Metabolic acidosis D. 3-phosphoglycerate
E. –

37
236. An infant has apparent D. Glycolysis
diarrhea resulting from improper E. Gluconeogenesis
feeding. One of the main diarrhea
effects is plentiful excretion of sodium 240. When blood circulation in the
bicarbonate. What form of acid-base damaged tissue is restored, then
balance disorder is the case? lactate accumulation comes to a stop
A. Respiratory alkalosis and glucose consumption decelerates.
B. Respiratory acidosis These metabolic changes are caused
C. No disorders of acid-base by activation of the following process:
balance will be observed A. Gluconeogenesis
D. Metabolic acidosis B. Aerobic glycolysis
E. Metabolic alkalosis C. Glycogen biosynthesis
D. Lipolysis
237. Buffer capacity of blood was E. Anaerobic glycolysis
decreased in the worker due to
exhausting muscular work. Entry of 241. A 48 year old patient
what acid substance to the blood can complained about intense pain, slight
this state be explained? swelling and reddening of skin over the
A. 1,3-bisphosphoglycerate joints, temperature rise up to 38oC.
B. Alpha-ketoglutarate Blood analysis revealed high
C. 3-phosphoglycerate concentration of urates. This condition
D. Lactate might be caused by disturbed
E. Pyruvate metabolism of:
A. Collagen
238. Analysis of blood serum of a B. Pyrimidines
patient revealed increase of alanine C. Cholesterol
aminotransferase and aspartate D. Carbohydrates
aminotransferase level. What E. Purines
cytological changes can cause such a
situation? 242. A 2-year-old child with mental
A. Disturbance of genetic and physical retardation has been
apparatus of cells delivered to a hospital. He presents
B. Cellular breakdown with frequent vomiting after having
C. Disorder of enzyme systems of meals. There is phenylpyruvic acid in
cells urine. Which metabolism abnormality is
D. Disturbance of cellular the reason for this pathology?
interrelations A. Phosphoric calcium metabolism
E. Disturbed function of energy B. Water-salt metabolism
supply of cells C. Amino-acid metabolism
D. Carbohydrate metabolism
239. Untrained people often have E. Lipidic metabolism
muscle pain after sprints as a result of
lactate accumulation. This might be 243. An infant has pylorospasm,
caused by intensification of the weakness, hypodynamia, convulsions
following biochemical process: as a result of frequent vomiting. What
A. Lipogenesis kind of acid-base disbalance is it?
B. Glycogenesis A. Excretory acidosis
C. Pentose phosphate pathway B. Metabolic acidosis

38
C. Excretory alkalosis 247. A patient has got frequent
D. Gaseous alkalosis internal organs' and mucosal
E. Exogenous nongaseous hemorrhages. The analysis indicated
acidosis insufficiency of hydroxyproline and
hydroxylysine in the collagenous fibres.
244. A 46-year-old patient The insufficiencuy of what vitamin
consulted a doctor complaining about caused the impairment of
joint pain that becomes stronger the hydroxylysation of abovementioned
day before the weather changes. aminoacids in a patient?
Blood examination revealed an A. Vitamin Н
increased concentration of uric acid. B. Vitamin К
The most probable cause of the C. Vitamin С
disease is the intensified D. Vitamin РР
disintegration of the following E. Vitamin А
substance:
A. Uridine triphosphate 248. A woman, who was keeping
B. Adenosine monophosphate a diet for a long time and eating refined
C. Thymidine monophosphate rice, has developed polyneuritis
D. Uridine monophosphate (beriberi disease). The absence of what
E. Cytidine monophosphate vitamin in the diet causes the
development of such disease?
245. A newborn child with A. Ascorbic acid
pylorostenosis has often repeating B. Pyridoxal
vomiting accompanied by apathy, C. Riboflavin
weakness, hypertonicity, sometimes D. Folic acid
convulsions. What disorder form of E. Thiamine
acid-base balance is it?
A. Metabolic acidosis 249. A 35-year-old woman with
B. Excretory acidosis chronic renal disease has developed
C. Nongaseous alkalosis osteoporosis. Deficiency of which from
D. Gaseous acidosis the belowmentioned substances
E. Gaseous alkalosis causes such complication?
A. 25OHD3
15. Vitamine Disorders B. Cholesterol
C. D2
D. D3
246. Pain along large nervous
E. 1.25(OH)2 D3
stems and increased amount of
pyruvate in the blood were revealed in
250. A 43-year-old man suffers
the patient. Insufficiency of what
from chronic atrophic gastritis and
vitamin can cause such change?
megaloblastic hyperchromic anemia.
A. Pantothenic acid
He also has methylmalonic aciduria.
B. В1
Insufficiency of what vitamin led to the
C. РР
development of such complex of
D. Biotin
symptoms?
E. В2
A. Vitamin В3
B. Vitamin В12
C. Vitamin В2

39
D. Vitamin В1 D. Vitamin C
E. Vitamin В5 E. Vitamin B1

251. A patient with symmetric 255. Hydroxylation of endogenous


dermatitis of open dermal areas came substrates and xenobiotics requires a
to the doctor. Talking to the patient the donor of protons. Which of the following
doctor found out that the patient vitamins can play this role?
mostly eats cereals and eats little A. Vitamin C
meat, milk and eggs. The deficiency of B. Vitamin E
which of the mentioned vitamins is C. Vitamin P
dominant in this patient? D. Vitamin A
A. Folic acid E. Vitamin B6
B. Tocopherol
C. Calciferol 256. Most participants of Magellan
D. Nicotinamide expedition to America died from
E. Biotin avitominosis. This disease declared
itself by general weakness,
252. There is an inhibited subcutaneous hemmorhages, falling of
coagulation in the patients with bile teeth, gingival hemmorhages. What is
ducts obstruction, bleeding due to the the name of this avitiminosis?
low level of absorbtion of a vitamin. A. Biermer's anemia
What vitamin is in deficiency? B. Pellagra
A. К C. Rachitis
B. Е D. Polyneuritis (beriberi)
C. D E. Scurvy
D. А
E. Carotene 257. Increased fragility of vessels,
enamel and dentine destruction
253. A 2-year-old child has got resulting from scurvy are caused by
intestinal dysbacteriosis, which results disorder of collagen maturation. What
in hemorrhagic syndrome. What is the stage of procollagen modification is
most likely cause of hemorrhage of disturbed under this avitaminosis?
the child? A. Glycosylation of hydroxylysine
A. Activation of tissue residues
thromboplastin B. Hydroxylation of proline
B. PP hypovitaminosis C. Detaching of N-ended peptide
C. Fibrinogen deficiency D. Formation of polypeptide chains
D. Vitamin K insufficiency E. Removal of C-ended peptide from
E. Hypocalcemia procollagen

254. Pyruvate concentration in the 258. Examination of a man who


patient’s urine has increased 10 times hadn't been consuming fats but had
from normal amount. What vitamin been getting enough carbohydrates
deficiency can be the reason of this and proteins for a long time revealed
change: dermatitis, poor wound healing, vision
A. Vitamin B6 impairment. What is the probable cause
B. Vitamin A of metabolic disorder?
C. Vitamin E A. Lack of oleic acid

40
B. Lack of linoleic acid, vitamins A, A. A
D, E, K B. C
C. Lack of vitamins PP, H C. B1
D. Lack of palmitic acid D. K
E. Low caloric value of diet E. B2

259. A woman who has been 263. A patient consulted a doctor


keeping to a clean-rice diet for a long about symmetric dermatitis of open skin
time was diagnosed with polyneuritis areas. It was found out that the patient
(beriberi). What vitamin deficit results lived mostly on cereals and ate too little
in development of this disease? meat, milk and eggs. What vitamin
A. Folic acid deficiency is the most evident?
B. Riboflavin A. Calciferol
C. Pyridoxine B. Folic acid
D. Thiamine C. Nicotinamide
E. Ascorbic acid D. Biotin
E. Tocopherol
260. A patient underwent an
operation on account of gall bladder 264. A patient suffers from vision
excision that resulted in obstruction of impairment - hemeralopy (night
Ca absorption through the bowels blindness). What vitamin preparation
wall. What vitamin wil stimulate this should be administered the patient in
process? order to restore his vision?
A. D3 A. Pyridoxine
B. B12 B. Retinol acetate
C. K C. Vicasol
D. PP D. Thiamine chloride
E. C E. Tocopherol acetate

261. Surgical removal of a part of 265. A 3 year old child with


stomach resulted in disturbed symptoms of stomatitis, gingivitis and
absorption of vitamin B12, it is excreted dermatitis of open skin areas was
with feces. The patient was diagnosed delivered to a hospital. Examination
with anemi A. What factor is revealed inherited disturbance of
necessary for absorption of this neutral amino acid transporting in the
vitamin? bowels. These symptoms were caused
A. Pepsin by the deficiency of the following
B. Folic acid vitamin:
C. Hydrochloric acid A. Niacin
D. Gastrin B. Cobalamin
E. Gastromucoprotein C. Vitamin A
D. Biotin
262. As a result of posttranslative E. Pantothenic acid
modifications some proteins taking
part in blood coagulation, particularly 266. Increased breaking of vessels,
prothrombin, become capable of enamel and dentine destruction in
calcium binding. The following vitamin scurvy patients are caused by disorder
takes part in this process: of collagen maturing. What stage of

41
modification of procollagen is that vitamin B6 is a componet of the
disordered in this avitaminosis? following enzyme:
A. Hydroxylation of proline A. Netoglubarate dehydromine
B. Formation of polypeptide chains B. Pyruvate dehydrostase
C. Glycosylation of hydroxylysine C. Glutamate decarboxylase
residues D. Aminolevulinate synthase
D. Removal of C-ended peptide E. Glycogen phosphorylase
from procollagen
E. Detaching of N-ended peptide 16. Disorders of
267. A 10-year-old girl often Termoregulation. Fever.
experiences acute respiratory
infections with multiple spotty 271. What heat transfer mechanism
haemorrages in the places of clothes is the most effective while the man
friction. Hypovitaminosis of what being at 80% of air moisture and the
vitamin is present at the girl? temperature +350С?
A. В6 A. Radiation
B. В1 B. Heat conduction
C. А C. Convection
D. В2 D. Evaporation
E. С E. --

268. Vitamin A deficit results in the 272. A 50-year-old patient with


impairment of twilight vision. Name typhoid fever was treated with
the cells that have the above- Levomycetin, the next day his condition
mentioned photoreceptor function: became worse, temperature rised to
A. Ganglion neurocytes 39,60С. What caused worthening?
B. Horizontal neurocytes A. Secondary infection addition
C. Bipolar neurons B. Reinfection
D. Cone receptor cells C. The effect of endotoxin agent
E. Rod receptor cell D. Allergic reaction
E. Irresponsiveness of an agent to
269. A sportsman was the levomycetin
recommended to take a medication
that contains carnitine in order to 273. A patient in winter fall down into
improve his results. What process is ice-hole, froze and fall ill. The
activated by carnitine the most? temperature increased up to 39,70С
A. Synthesis of steroid hormones and ranged from 39,00C tо 39,80С .
B. Tissue respiration Name the type of temperature curve of
C. Synthesis of ketone bodies this patient.
D. Fatty acids transport to A. Febris recurrens
mitochondrions B. Febris continua
E. Synyhesis of lipids C. Febris intermittens
D. Febris remittens
270. A newborn child has E. Febris hectica
convulsions that have been observed
after prescription of vitamin B6. This 274. A patient experienced a sudden
most probable cause of this effect is temperature rise up to 390С. After 6

42
hours the temperature normalized. On C. Violation of heat loss through
the 2-nd day the attack recurred: in lungs
the period of paroxysm the D. Inflammatory barrier formation in
0
temperature reached 41 С, apyrexial injection place
period began after 8 hours. What type E. Violation of heat-producing
of temperature profile is it? mechanisms
A. Recurrent
B. Hectic 278. A lightly dressed man is
C. Septic standing in a room, air temperature is
D. Continued +14°C, windows and doors are closed.
E. Intermitting In what way does he emit heat the most
actively?
275. A patient who suffers from A. Heat radiation
pneumonia has high body temperatur B. Heat conduction
e. What biologically active substance C. Evaporation
plays the leading part in origin of this D. Perspiration
phenomenon? E. Convection
A. Interleukin-I
B. Bradykinin 279. A patient who had been
C. Histamine working hard under conditions of
D. Leukotrienes elevated temperature of the
E. Serotonin environment, has now a changed
quantity of blood plasma proteins. What
276. The patient has come to the phenomenon is the case?
hospital from the smelting workshop in A. Absolute hyperproteinemia
the condition of hyperthermia. What is B. Absolute hypoproteinemia
the direct cause of loss of C. Relative hyperproteinemia
consciousness at the heat stroke? D. Disproteinemia
A. Decreased brain blood supply E. Paraproteinemia
B. Arterial pressure drop
C. Increased water loss through 280. People adapted to high external
sweating temperatures have such pecularity:
D. Dilatation of peripheral vessels profuse sweating isn't accompanied by
E. Decrease of heart output loss of large volumes of sodium
chloride. This is caused by the effect of
277. A 52 year-old patient with the following hormone upon the
bronchial asthma was treated with perspiratory glands:
glucocorticoids. Fever reaction A. Aldosterone
appeared as a result of postinjective B. Natriuretic
abscess. The patient had subfebrile C. Cortisol
temperature, which didn’t correspond D. Tgyroxin
to latitude and severity of E. Vasopressin
inflammatory process. Why did patient
have low fever reaction? 281. A 25-year-old man has spent a
A. Inhibited endogen pyrogens long time in the sun under high air
production humidity. As a result of it his body
B. Thermoregulation center temperature rose up to 39oC. What
inhibition pathological process is it?

43
A. Hyperthermia what acid substance to the blood can
B. Burn disease this state be explained?
C. Hypothermia A. Lactate
D. Infectious fever B. α-ketoglutarate
E. Noninfectious fever C. Pyruvate
D. 1,3-bisphosphoglycerate
282. Which way of heat emission E. 3-phosphoglycerate
by the bodies of greenhouse workers
is the most effective at the 286. If strong oxidizers get into the
temperature of 36oC degrees and bloodstream, a methemoglobin is
relative humidity of 70%? formed. It is a compound, where iron
A. Heat radiation (II) becomes iron (III). What has to be
B. Convection done to save the patient?
C. Thermal conduction A. He has to be given pure oxygen
D. Liquid evaporation B. He has to be calmed down and
E. - put to bed
C. Respiratory centers have to be
283. After transfusion of 200 ml of stimulated
blood a patient presented with body D. Interchangeable hemotransfusion
temperature rise up to 37,9oC. Which has to be done
of the following substances is the E. Patient has to be exposed to the
most likely cause of temperature rise? fresh air
A. Interleukin-1
B. Tumour necrosis factor 287. A 56 y.o. patient has been
C. Interleukin-2 suffering from thyreotoxicosis for a long
D. Interleukin-3 time. What type of hypoxia can be
E. Interleukin-4 developed?
A. Hemic
284. A patient has been diagnosed B. Respiratory
with influenza. His condition became C. Mixed
drastically worse after taking D. Tissue
antipyretic drugs. His consciousness E. Circulatory
is confused, AP is 80/50 mm Hg, Ps is
140/m, body temperature droped 288. There is a severe time
down to 35,8oC. What complication restriction for people's staying at a
developed in this patient? height of over 800 m above the sea
A. Alkalosis level without oxygen bombs. What is
B. Collapse the life limiting factor in this case?
C. Hyperthermia A. Partial oxygen pressure
D. Acidosis B. Moisture level
E. Hypovolemia C. Temperature
D. Earth gravity
17. Hypoxia E. Ultraviolet intensity

285. Buffer capacity of blood was 18. Disorders of


decreased in the worker due to Respiratory System
exhausting muscular work. Entry of

44
289. Lung of premature infant is A. Apneustic respiration
presented on electronic B. Cheyne-Stockes respiration
photomicrography of biopsy material. C. Kussmaul respiration
Collapse of the alveolar wall caused D. Biot's respiration
by the deficiency of surfactant was E. Gasping respiration
revealed. Disfunction of what cells of
the alveolar wall caused it? 293. A patient because of
A. Fibroblasts pathological process has thickened
B. Alveolar macrophages alveolar membrane. What will be
C. Secretory cells reduced directly as a result this?
D. Alveocytes type II A. Oxygen capacity of blood
E. Alveocytes type I B. Alveolar lung ventilation
C. Diffuse lung capacity
290. Child asked you to puff up the D. Reserve expiratiory capacity
balloon as much as possible for a one E. Minute respiratory capacity
exhalation.What air volume will you
use? 294. A woman with myasthenia
A. Functional residual volume developed respiratory disorders, which
B. Inspiration volume required artificial ventilation. What type
C. Total volume of the lungs pf respiratory insufficiency did the
D. Backup volume of the inspiration woman have?
E. Vital volume of the lungs A. Centrogenic
B. Restrictive
291. While having the dinner the C. Neuromuscular
child choked and aspirated the food. D. Thoracophrenic
Meavy cough has started, skin and E. Obstructive
mucose are cyanotic, rapid pulse, rear
breathing, expiration is prolonged. 295. A man in a quiet condition
What disorder of the external has increased activity of the muscles,
breathing developed in the child? responsible for the inspiration. What
A. Biot's breathing from the belowmentioned can cause
B. Stage of expiratory dyspnea on this?
asphyxia A. Infrequent breathing
C. Alternating breathing B. Superficial breathing
D. Stage of inspiratory dyspnea on C. Airways narrowing
asphyxia D. Negative intrapleural pressure
E. Stenotic breathing E. Reduction of minute respiratory
capacity
292. A 62-year-old patient was
admitted to the neurological 296. During the examination of
department due to cerebral blood gases a patient with chronic
haemorrage. Condition is grave. respiratory diseases accompanied by
There is observed progression of dyspnea, tachycardia and cyanosis
deepness and frequency of breath demonstrated the development of
that turnes into reduction to hypoxemia and hypercapnia. What
apnoea,and the cycle repeates. What disorder of the external respiration does
respiration type has developed in the the patient have?
patient? A. Hyperperfusion

45
B. Hyperdiffusion B. Decrease of bicarbonate
C. Hyperventilation reabsorption in kidneys
D. Hypoperfusion C. Hyperventilation
E. Hypoventilation D. Intensification of acidogenesis
E. Hypoventilation
297. Intrapleural pressure is being
measured in a person. In what phase 301. Spasm of smooth muscle of
has a person hold his breath if the bronchi developed in the patient. Usage
pressure is - 25 cm H2O? of activators of what membrane
A. Speed up inspiration (Forced cytoreceptors is fisiologically valid to
inspiration ) decrease attack?
B. Calm expiration A. alpha- and beta-аdrenoreceptors
C. Calm inspiration B. Н-cholinoreceptors
D. Speed up expiration C. Beta-adrenoreceptors
E. – D. Alpha-аdrenoreceptors
E. М-cholinoreceptors
298. While having the dinner the
child choked and aspirated the food. 302. Child asked you to puff up the
Heavy cough has started, skin and balloon as much as possible for a one
mucose are cyanotic, pulse is rapid, exhalation.What air volume will you
respiration is infrequent, expiration is use?
prolonged. What disorder of the A. Vital volume of the lungs
external respiration has the child? B. Inspiration volume
A. Biot's respiration C. Functional residual volume
B. Stenotic respiration D. Total volume of the lungs
C. Stage of expiratory dyspnea on E. Backup volume of the inspiration
asphyxia
D. Alternating respiration 303. The alveolar ventilation of the
E. Stage of inspiratory dyspnea on patient is 5 L/min, the breath frequency
asphyxia is 10 per/min, and the tidal volume is
700 ml. What is the patient's dead
299. A man's intrapleural pressure space ventilation?
is being measured. In what phase did A. 0,7 L/min
the man hold his breath, it his B. 1,0 L/min
pressure is 7.5 cm Hg? C. 2,0 L/min
A. Quiet expiration B. 4,3 L/min
B. Forced inspiration E. -
C. Quiet inspiration
D. Forced expiration 304. A patient after pathological
E. – process has a thickened alveolar
membrane. The direct consequence of
300. A group of mountain climbers the process will be the reduction of:
went through the blood analysis at the A. Reserve expiratiory capacity
height of 3000 m. It revealed decrease B. Oxygen capacity of blood
of HCO3 to 15 micromole/l (standard is C. Minute respiratory capacity
22-26 mmole/l). What is the D. Alveolar lung ventilation
mechanism of HCO3, decrease? E. Diffuse lung capacity
A. Decrease of ammoniogenesis

46
305. X-ray examination discovered
lungs emphysema in the patient. What 309. A 12 y.o. boy who suffers from
is the reason of short breath bronchial asthma has an acute attack
development in this case? of asthma: evident expiratory dyspnea,
A. Decreased lungs elasticity skin pallor. What type of alveolar
B. Decreasing of alveoli receptors ventilation disturbance is it?
sensitivity A. Throracodiaphragmatic
C. Increased lungs elasticity B. Central
D. Inhibition of respiratory center C. Obstructive
E. Excitation of respiratory center D. Restrictive
E. Neuromuscular
306. Part of alveoles of a preterm
infant didn't spread because of 310. Lungs of a preterm infant have
enhanced elastic recoil of lungs. How areas of atelectasis (pulmonary
can this recoil be reduced? collapse). The main cause is:
A. By pure oxygene inhalation A. Underdeveloped inspiration
B. By surfactant introduction muscles
C. By glycose introduction B. Diminished force of surface
D. By artificial pulmonary tension of lungs
ventilation C. Increased viscous resistance
E. By fluid suction from the D. Surfactant excess
respiratory tracts E. Surfactant deficiency

307. A patient has a transverse 311. An unconscious young man


disruption of spinal cord below the IV with signs of morphine poisoning
thoracic segment. What changes of entered admission office. His
respiration will it cause? respiration is shallow and infrequent
A. Respiration will stay unchanged which is caused by inhibition of
B. Respiration will become less respiratory centre. What type of
frequent respiratory failure is it?
C. Respiration will become more A. Perfusive
frequent B. Ventilative restrictive
D. Respiration will become deeper C. Diffusive
E. Respiration will stop D. Ventilative obstructive
E. Ventilative dysregulatory
308. Examination of a miner
revealed pulmonary fibrosis 312. A young woman who entered a
accompanied by disturbance of production department where it strongly
alveolar ventilation. What is the main smelt of paints and varnishes had a
mechanism of this disturbance? bronchospasm. This reflex was caused
A. Constriction of superior by irritation of the following receptors:
respiratory tracts A. Central chemoreceptors
B. Limitation of respiratory surface B. Peripheral chemoreceptors
of lungs C. Pleura receptors
C. Disturbance of neural respiration D. Irritant
control E. Juxtaglomerular
D. Bronchi spasm
E. Limitation of breast mobility

47
313. Voluntary breath-holding 19. Disorders of Blood.
caused increase of respiration depth
and frequency. The main factor RBC Related Pathology
stimulating these changes of external
respiration is: 317. A 32-year-old patient was
A. Increased tension of CO2 in admitted to the hospital with gross
blood bloodloss due to auto accident trauma.
B. Decreased tension of O2 in Ps – 110 Bpm, RR - 22 pm, BP-
blood 100/60mm Hg. What changes in the
C. Decreased concentration of H+ blood will occur in an hour after the
in blood bloodloss?
D. Increased tension of O2 in blood A. Hypovolemia
E. Decreased tension of CO2 in B. Hypoproteinemia
blood C. Hypochromia of erythrocytes
314. A patient staying in the D. Leukopenia
pulmonological department was E. Erythropenia
diagnosed with pulmonary
emphysema accompanied by reduced 318. Blood sampling for bulk
elasticity of pulmonary tissue. What analysis is recommended to be
type of respiration is observed? performed on an empty stomack and in
A. Inspiratory dyspnea the morning. What changes in blood
B. Periodic respiration count can occur if to perform blood
C. Superficial respiration sampling after food intake?
D. Infrequent respiration A. Increased plasma proteins
E. Expiratory dyspnea B. Increased contents of leukocytes
C. Increased contents of
315. A patient with marked erythrocytes
pneumofibrosis that developed after D. Reduced contents of
infiltrating pulmonary tuberculosis has thrombocytes
been diagnosed with respiratory E. Reduced contents of erythrocytes
failure. What is its pathogenetic type?
A. Dysregulatory 319. In the blood of a 26-year-old
B. Obstructive man it was revealed 18% of
C. Reflex erythrocytes of the spherical, ball-
D. Apneistic shaped, flat and thorn-like shape. Other
E. Restrictive eritrocytes were in the form of the
concavo-concave disks. How is such
316. Lung ventilation in a person is phenomenon called?
increased as a result of physical A. Pathological poikilocytosis
activity. Which of the following indices B. Physiological anisocytosis
of the external respiration is much C. Pathological anisocytosis
higher than in a state of rest? D. Erytrocytosis
A. Respiratory volume E. Physiological poikilocytosis
B. Expiratory reserve volume
C. Vital capacity of lungs 320. In the microspecimen of red
D. Inspiratory reserve volume bone marrow there were revealed
E. Total lung capacity multiple capillares through the walls of

48
which mature blood cells penetrated. 324. A denaturation of proteins can
What type of capillares is it? be found in some substances. Specify
A. Sinusoidal the substance that is used for the
B. Fenestrational incomplete denaturation of hemoglobin:
C. Somatical A. Sulfuric acid
D. Visceral B. Nitric acid
E. Lymphatic C. Toluene
D. Sodium hydroxide
321. A 38-year-old woman was E. Urea
admitted to the admission-diagnostic
department with uterine bleeding. 325. Examination of a pregnant
What are the most likely changes of woman revealed twice as much
blood? concentration of fibrinogen in blood
A. Increase of haematocrite rate plasm. What ESR can this woman
B. Leukopenia have?
C. Leucocytosis A. 0-5 mm/h
D. Polycythemia B. 5-10 mm/h
E. Reduction of haematocrite rate C. 2-12 mm/h
D. 40-50 mm/h
322. On blood grouping on the E. 10-15 mm/h
system ABO, standart serum of the I
and II groups caused erythrocytes 326. Blood count of an athlete is as
agglutination of the examined blood follows: erythrocytes - 5,5*1012/l, Hb-
and serum group of the III didn't. What 180 g/l, leukocytes – 7*109/l,
agglutinogens are in this neutrophils - 64%, basophils - 0,5%,
erythrocytes? eosinophils - 0,5%, monocytes - 8%,
A. А and В lymphocytes - 27%. First of all, such
B. С results indicate the stimulation of:
C. В A. Leukopoiesis
D. А B. Lymphopoiesis
E. D and C C. Immunogenesis
D. Erythropoiesis
323. If strong oxidizers get into the E. Granulocytopoiesis
bloodstream, a methemoglobin is
formed. It is a compound, where iron 327. Patient with hypochromic
(II) becomes iron (III). What has to be anemia has splitting hair and loss of
done to save the patient? hair, increased nail brittling and taste
A. He has to be given pure oxygen alteration. What is the mechanism of
B. He has to be calmed down and the development of these symptoms?
put to bed A. Decreased production of
C. Respiratory centers have to be parathyrin
stimulated B. Decreased production of thyroid
D. Interchangeable hormones
hemotransfusion has to be done C. Deficiency of vitamin В12
E. Patient has to be exposed to the D. Deficiency of vitamin А
fresh air E. Deficiency of iron-containing
enzymes

49
328. Substitution of the glutamic A. FADH2
acid on valine was revealed while B. FMNH2
examining initial molecular structure. C. Pyridoxalphosphate
For what inherited pathology is this D. Ubiquinone
typical? E. NADPH
A. Minkowsky-Shauffard disease
B. Hemoglobinosis 332. Patient 54 year-old, 5th day
C. Sickle-cell anemia after surgical operation. Blood count:
D. Favism Erythrocytes 3,6*1012/l, Hemoglobin 95
E. Thalassemia g/l, Erythrocyte’s hemoglobin content
(color index) 0,78; Leukocytes 16*109/l,
329. A 58-year-old woman Platelets 450*109/l Blood picture:
complaints of increased tiredness, anizocytosis, poikilocytosis,
decreased capasity for work, reticulocytes- 3,8%. What anemia does
somnolence and dyspnea during fast this patient have?
walking. Blood test revealed: A. Chronic posthemorragic anemia
12
erythrocytes - 4,6*10 /l, hemoglobin - B. Acquired hemolytic anemia
92 g/l, colour index - 0,6. Blood smear C. Acute posthemorragic anemia
demonstrated high contents of D. Anemia from iron deficiency
microcytes and anulocytes. What E. Hypoplastic anemia
anemia is it typical for?
A. Acute posthemorrhagic 333. In the blood of a 26-year-old
B. Hemolytic man 18% of erythrocytes of the
C. Iron deficiency spherical, ball-shaped, flat and thorn-
D. Penicious like shape have been revealed. Other
E. Sickle cell eritrocytes were in the form of the
concavo-concave disks. How is this
330. A 43-year-old man suffers phenomenon called?
from chronic atrophic gastritis and A. Physiological poikilocytosis
megaloblastic hyperchromic anemia. B. Erytrocytosis
He also has methylmalonic aciduria. C. Pathological poikilocytosis
Insufficiency of what vitamin led to the D. Physiological anisocytosis
development of such complex of E. Pathological anisocytosis
symptoms?
A. Vitamin В3 334. A patient's blood was analyzed
B. Vitamin В12 and the decreased erythrocyte’s
C. Vitamin В2 sedimentation rate (ESR) was
D. Vitamin В1 discovered. What disease from the
E. Vitamin В5 listed below is accompanied with
decreased ESR?
331. A 38-year-old patient after A. Myocardial infarction
taking aspirin and sulphanilamides B. Hepatitis
has profound hemolysis of C. Splenomegaly
erythrocytes, which is caused by D. Polycytemia
glucose 6-phosphate dehydrogenase E. Vitamin B deficiency
insufficiency. The disorder of
formation of what coenzyme causes 335. A 7-year-old girl has signs of
this pathology? anemia. Laboratory examination

50
revealed pyruvate kinase deficiency in 339. A 56 year old patient came to a
erythrocytes. What process hospital with complaints about general
disturbance plays the main role in weakness, tongue pain and burning,
anemia development? sensation of limb numbness. In the past
A. Anaerobic glycolysis he underwent resection of forestomach.
B. Tissue respiration In blood: Hb-80 g/l; erythrocytes -
C. Oxidative phosphorylation 2,0*1012/l; colour index - 1,2, leukocytes
D. Aminoacids desamination - 3,5*109/l. What anemia type is it?
E. Peroxide decomposition A. Iron-deficient
B. Hemolytic
336. A 38-year-old woman was C. Aplastic
admitted to the admission-diagnostic D. B12-folate deficient
department with uterine bleeding. E. Posthemorrhagic
What are the most likely changes of
blood? 340. 2 years ago a patient
A. Reduction of haematocrite rate underwent resection of pyloric part of
B. Increase of haematocrite rate stomach. He complains of weakness,
C. Leucocytosis periodical dark shadows beneath his
D. Leukopenia eyes, dyspne A. In blood: Hb - 70 g/l,
E. Polycythemia erythrocytes - 3,0*1012/l, colour index -
0,7. What changes of erythrocytes in
337. Examination of a 43 y.o. blood smears are the most typical for
anephric patient revealed anemia this condition?
symptoms. What is the cause of these A. Megalocytes
symptoms? B. Ovalocytes
A. Folic acid deficit C. Schizocytes
B. Vitamin B12 deficit D. Macrocytes
C. Reduced synthesis of E. Microcytes
erythropoietins
D. Enhanced destruction of 341. A 20 year old patient complains
erythrocytes of general weakness, dizziness, quick
E. Iron deficit fatigability. Blood analysis results: Hb-
80 g/l. Microscopical examination
338. In some regions of South results: erythrocytes are of modified
Africa there is a spread sickle-shaped form. This condition might be caused
cell anemia, in which erythrocytes by:
have shape of a sickle as a result of A. Obturative jaundice
substitution of glutamin by valine in B. Addison's disease
the hemoglobin molecul e. What is the C. Hepatocellular jaundice
cause of this disease? D. Acute intermittent porphyria
A. Gene mutation E. Sickle-cell anemia
B. Disturbance of mechanisms of
genetic information realization 342. A 55 y.o. woman consulted a
C. Genomic mutations doctor about having continuous cyclic
D. Crossingover uterine hemorrhages for a year,
E. Transduction weakness, dizziness. Examination
revealed skin pallor. Hemogram: Hb –
70 g/l, erythrocytes - 3.2 1012/l, color

51
index - 0.6, leukocytes – 6.0 109/I, C. Enzymopathy
reticulocytes - 1%; erythrocyte D. Membranopathy
hypochromia. What anemia is it? E. Hemoglobinopathy
A. B12-folate-deficiency anemia
B. Aplastic anemia 20. Disorders of Blood.
C. Iron-deficiency anemia
D. Chronic posthemorrhagic WBC Related Pathology
anemia
E. Hemolytic anemia 346. On microscopic examination of
the enlarged neck gland of a 14-year-
343. A woman with III (B), Rh– old girl it was revealed destruction of
blood group born a child with II (A) the tissue structure of the node,
blood group. The child is diagnosed absence of the lymph follicles, sclerotic
with hemolytic disease of newborn as and necrosis parts, cell constitution of
a result of rhesus incompatibility. the node is polymorphous, lymphocites,
What blood group is the child's father eosinophiles, atypical cells of the large
likely to have? size with multiple-lobule nuclei
A. II (A), Rh+ (Beresovsky-Shternberg cells) and
B. II (A), Rh- onenucleus cells of the large size are
C. (0), Rh+ present. What is the most likely
D. Ill (B), Rh+ diagnosis?
E. I (0), Rh- A. Fungous mycosis
B. Chronic lympholeucosis
344. A 25 year old Palestinian C. Lymphogranulomatous
woman complains of weakness, D. Berkitt's lymphoma
dizziness, dyspne A. In anamnesis: E. Acute lympholeucosis
periodically exacerbating anemi A. In
blood: Hb - 60 g/l, erythrocytes - 347. A patient with acute
2,5*1012/l, reticulocytes - 35‰, myeloblast leucosis has developed liver
anisocytosis and poikilocytosis of and spleen enlargement, anemia,
erythrocytes, a lot of target cells and myeloblasts in peripheral blood. What
polychromatophils. What type of principal sign allows to differ myeloblast
anemia is it? leukosis from chronic one?
A. Sickle-cell anemia A. Leukemic collapse
B. Addison-Biermer disease B. Pancytopenia
C. Glucose 6-phosphate C. Anemia
dehydrogenase-deficient anemia D. Blast cells in peripheral blood
D. Minkowsky-Shauffard disease E. Thrombocytopenia
E. Thalassemia
348. A 16-year-old boy was
345. A 34 year old woman was performed an appendectomy. He has
diagnosed with hereditary been hospitalized for right lower
microspherocytic hemolytic anemia quadrant abdominal pain within 18
(Minkowsky-Shauffard disease). What hours. The surgical specimen is
mechanism caused haemolysis of edematous and erythematous.
erythrocytes? Infiltration by what of the following cells
A. Autoimmune disorder is the most typical for the process
B. Bone marrow hypoploasia occuring here?

52
A. Basophils C. Regenerative left shift
B. Eosinophils D. Right shift
C. Monocytes E. Degenerative left shift
D. Neutrophils
E. Limphocytes 352. 24 hours after appendectomy
blood of a patient presents neutrophilic
349. In the blood of a 26-year-old leukocytosis with regenerative shift.
man 18% of erythrocytes of the What is the most probable mechanism
spherical, ball-shaped, flat and thorn- of leukocytosis development?
like shape have been revealed. Other A. Amplification of leukopoiesis and
erythrocytes were in the form of the decelerated emigration of leukocytes to
concavo-concave disks. How is this the tissues
phenomenon called? B. Decelerated leukocyte destruction
A. Physiological poikilocytosis C. Deceleratied emigration of
B. Erytrocytosis leukocytes to the tissues
C. Pathological poikilocytosis D. Amplification of leukopoiesis
D. Physiological anisocytosis E. Redistribution of leukocytes in the
E. Pathological anisocytosis organism

350. Blood sampling for bulk 353. A 23 y.o. patient complains of


analysis is recommended to be weakness, temperature rise up to 38-
performed on an empty stomack and 400C. Objectively: liver and spleen are
in the morning. What changes in blood enlarge d. Hemogram: Hb- 100 g/l,
composition can occur if to perform erythrocytes - 2,9*1012/l, leukocytes -
blood sampling after food intake? 4,4*109/l, thrombocytes – 48*109/l,
A. Reduced contents of segmentonuclear neutrophils - 17%,
erythrocytes lymphocytes - 15%, blast cells - 68%.
B. Increased contents of All cytochemical reactions are negativ
erythrocytes e. Make a hematological conclusion:
C. Increased contents of A. Acute erythromyelosis
leukocytes B. Acute myeloblastic leukosis
D. Increased plasma proteins C. Chronic myeloleukosis
E. Reduced contents of D. Undifferentiated leukosis
thrombocytes E. Acute lymphoblastic leukosis

351. A patient operated on 354. Two hours after an exam a


complicated appendicitis has the student had a blood count done and it
following changes of blood count: was revealed that he had leukocytosis
erythrocytes - 4,0.1012/l, Нb - 120 g/l, without significant leukogram
color index - 0,9, leukocytes – modifications. What is the most
18.109/l, basophils - 0, eosinophils - 0, probable mechanism of leukocytosis
myelocytes - 0, juvenile - 0, stab development?
neutrophils - 20, segmentonuclear A. Deceleration of leukocyte
neutrophils - 53, lymphocytes - 21, migration to the tissues
monocytes - 5. How is such nuclear B. Deceleration of leukocyte lysis
shift of leukocytic formula called? C. Leukopoiesis intensification and
A. Hyperregenerative deceleration of leukocyte lysis
B. Regeneratively-degenerative D. Leukopoiesis intensification

53
E. Redistribution of leukocytes in 358. Having helped to eliminate
the organism consequences of a failure at a nuclear
power plant, a worker got an irradiation
355. Parents of a 3 year old child doze of 500 roentgen. He complains of
have been giving him antibiotics with headache, nausea, dizziness. What
purpose of preventing enteric changes in leukocytes quantity can be
infections for a long time. A month expected 10 hours after irradiation?
later the child's condition changed for A. Lymphocytosis
the worse. Blood examination B. Neutrophilic leukocytosis
revealed apparent leukopenia and C. Leukopenia
granulocytopenia. What is the most D. Leukemia
probable mechanism of blood E. Agranulocytosis
changes?
A. Myelotoxic 359. 24 hours after appendectomy
B. Age-specific blood of a patient presents neutrophilic
C. Redistributive leukocytosis with regenerative shift.
D. Autoimmune What is the most probable mechanism
E. Hemolytic of leukocytosis development?
A. Deceleratied emigration of
356. A patient has a cluster of leukocytes to the tissues
matted together dense lymph nodes B. Amplification of leukopoiesis
on his neck. Histological examination C. Redistribution of leukocytes in the
of a removed lymph node revealed organism
proliferation of reticular cells, D. Amplification of leukopoiesis and
presense of Reed-Sternberg cells. decelerated emigration of leukocytes to
What disease is meant? the tissues
A. Lymphogranulomatosis E. Decelerated leukocyte destruction
B. Myeloblastic leukosis
C. Lymphocytic leukosis 360. A 47 year old man with
D. Lymphoblastic leukosis myocardium infarction was admitted to
E. Myelocytic leukosis the cardiological department. What
changes of cellular composition of
357. A 26 year old man is in the peripheral blood are induced by
torpid shock phase as a result of a car necrotic changes in the myocardium?
accident. In blood: 3,2*109/l. What is A. Monocytosis
the leading mechanism of leukopenia B. Eosinophilic leukocytosis
development? C. Neutrophilic leukocytosis
A. Redistribution of leukocytes in D. Lymphopenia
bloodstream E. Thrombocytopenia
B. Leikopoiesis inhibition
C. Lysis of leukocytes in the blood- 361. A 5 year old child is ill with
forming organs measles. Blood analysis revealed
D. Intensified elimination of increase of total number of leukocytes
leukocytes from the organism up to 13*109/l. Leukogram: basophils -
E. Disturbed going out of mature 0, eosinophils - 1, myelocytes - 0,
leukocytes from the marrow into the juvenile neutrophils - 0, band
blood neutrophils - 2, segmented neutrophils -

54
41, lymphocytes - 28, monocytes - 28. D. Hageman's factor
Name this phenomenon: E. Rosental's factor
A. Lymphocytosis
B. Agranulocytosis 365. A 2-year-old child has got
C. Eosinopenia intestinal dysbacteriosis, which results
D. Monocytosis in hemorrhagic syndrome. What is the
E. Neutropenia most likely cause of hemorrhage of this
child?
21. Disorders of A. Vitamin К insufficiency
B. Hypocalcemia
Hemostasis C. РР hypovitaminosis
D. Fibrinogen deficiency
362. A 43-year-old patient has E. Activation of tissue
thrombopenia, reduction of thromboplastin
fibrinogen, products of degradation of
fibrin presented in the blood, petechial 366. A patient with liver disease
haemorrhage along with septic shock. revealed the decreasing of prothrombin
What is the most likely cause of the level in the blood. It can, first of all,
changes? result in the impairment of:
A. Autoimmune thrombocytopenia A. The first phase of the coagulatory
B. DIC-syndrom hemostasis
C. Exogenous intoxication B. The second phase of the
D. Disorder of thrombocytes coagulatory hemostasis
production C. Vascular-thrombocytic
E. Haemorrhagic diathesis hemostasis
D. Anticoagulative properties of the
363. Punctata hemorrhage was blood
found out in the patient after E. Fibrinolysis
application of a tourniquet. With
disfunction of what blood cells is it 367. There is an inhibited
connected? coagulation in the patients with bile
A. Eosinophiles ducts obstruction, bleeding due to the
B. Lymphocytes low level of absorbtion of a vitamin.
C. Monocytes What vitamin is in deficiency?
D. Neutrophiles A. К
E. Platelets B. Е
C. D
364. A 6-months-old baby has D. А
got frequent and extensive subdermal E. Carotene
hemorrhages. The administration of
the synthetic analogue of vitamin K 368. Punctata hemorrhage was
(vicasol) was effective. γ-carboxylation found out in the patient after application
of glutamic acid of what protein of of a tourniquet. With disfunction of what
blood coagulation system does this blood cells is it connected?
vitamin take part in? A. Monocytes
A. Antihemophilic globulin A B. Eosinophiles
B. Fibrinogen C. Neutrophiles
C. Prothrombin D. Lymphocytes

55
E. Platelets D. Vitamin E
E. Vitamin C
369. A couple came for medical
genetic counseling. The man has 373. As a result of posttranslative
hemophilia, the woman is healthy and modifications some proteins taking part
there were no cases of hemophilia in in blood coagulation, particularly
her family. What is the risk of having a prothrombin, become capable of
sick child in this family? calcium binding. The following vitamin
A. 25% takes part in this process:
B. 0 A. A
C. 100% B. C
D. 75% C. B1
E. 50% D. K
E. B2
370. A patient with tissue trauma
was taken a blood sample for the 374. A 16 year old boy after an
determination of blood clotting illness has diminished function of
parameters. Specify the right protein synthesis in liver as a result of
sequence of extrinsic pathway vitamin K deficiency. It will cause
activation. disturbance of:
A. III – VIII: TF – Xa A. Erythropoietin secretion
B. III – VIIa – Xa B. Osmotic blood pressure
C. III – IV – Xa C. Blood coagulation
D. IV – VIII: TF – Xa D. Anticoagulant generation
E. IV – VIIa – Xa E. Erythrocyte sedimentation rate

371. A patient was ill with burn 375. A clinic observes a 49 year old
disease that was complicated by DIC patient with significant prolongation of
syndrom e. What stage of DIC coagulation time, gastrointestinal
syndrome can be suspected if it is haemorrhages, subcutaneous
known that the patient's blood hematomas. These symptoms might be
coagulates in less than 3 minutes? explained by the deficiency of the
A. Hypercoagulation following vitamin:
B. Transition phase A. E
C. Fibrinolysis B. H
D. Terminal C. B6
E. Hypocoagulation D. B1
E. K
372. Examination of a patient with
frequent hemorrhages from internals 376. A tooth extraction in a patient
and mucous membranes revealed with chronic persistent hepatitis was
proline and lysine being a part of complicated with prolonged
collagen fibers. What vitamin absence hemorrhage. What is the reason for the
caused disturbance of their haemorrhagic syndrome?
hydroxylation? A. Decrease in thrombin production
A. Vitamin A B. Fibrinolysis intensification
B. Thiamine C. Increase in thromboplastin
C. Vitamin K production

56
D. Increase in fibrinogen synthesis 380. In a 45-year-old patient on ECG
E. Decrease in fibrin production it was revealed: sinus rhythm, the
number of auricular complexesexceeds
377. A 10-year-old girl has a history number of ventricular complexes;
of repeated acute respiratory viral progressing extension of the P-Q
infection. After recovering she interval from complex to complex;
presents with multiple petechial fallout of some ventricular complexes;
hemorrhages on the sites of friction Р waves and QRST complexes are
from clothing rubbing the skin. What without changes. Name the type of
kind of hypovitaminosis has this girl? heart rhythm disfunction.
A. B2 A. Complete atrioventricular block
B. B1 B. Atrioventricular block of the II
C. C degree
D. B6 C. Atrioventricular blockade of the I
E. A degree
D. Synoauricular block
22. Disorders of E. Intraatrial block

Cardiovascular System 381. While emotional excitement the


heart rate in a 30-year-old person run
378. Processes of repolarisation up to 112 Bpm. What part of the
are disturbed in ventricular conducting system of the heart caused
myocardium in examined person. It it?
will cause amplitude abnormalities of A. Intraventricular node
configuration and duration of the B. Synoatrial node
wave: C. His bundle branches
A. R D. His bundle
B. P E. Purkinje's fibers
C. Q
D. S 382. Dystrophic changes of the heart
E. Т muscle are accompanied with cardiac
cavity enlargement, decrease of the
379. The patient with acute strength of heart contraction, increased
miocardial infarction was given amount of blood, which remains in the
intravenously different solutions during heart during systolic phase, overfilled
8 hours with medical dropper 1500ml veins. For what state of heart is it
and oxygen intranasally. He died characteristic?
because of pulmonary edema. What A. Tonogenic dilatation
caused the pulmonary edema? B. Emergency stage of
A. Inhalation of the oxygen hyperfunction and hypertrophy
B. Decreased oncotic pressure due C. Myogenic dilatation
to hemodilution D. Cardiosclerosis
C. Neurogenic reaction E. Tamponage of the heart
D. Allergic reaction
E. Volume overload of the left 383. Transmural myocardial
ventricular infarction in the patient was
complicated with progressive acute left

57
ventricle insufficiency. What is the E. Resorption of enzymes from
most typical for this state? necrotized area of myocardium
A. Edema of the extremities
B. Cyanosis 387. The high level of Lactate
C. Edema of the lungs Dehydrogenase (LDH) isozymes
D. Arterial hypertension concentration showed the increase of
E. Ascites LDH-1 and LDH-2 in a patient’s blood
plasma. Point out the most probable
384. A 48-year-old patient after diagnosis:
severe psychoemotional exertion A. Diabetes mellitus
suddenly began feeling sharp pain in B. Skeletal muscle dystrophy
the heart region, irradiating into left C. Viral hepatitis
arm. Nitroglycerin releaved pain 10 D. Myocardial infarction
minutes later. What pathogenetic E. Acute pancreatitis
mechanism is responsible for the
development of pain in this case? 388. After the trauma, the patient’s
A. Compression of coronary right n.vagus was damaged. Which
vessels violation of the cardiac activity is
B. Spasm of coronary vessels possible in this case?
C. Dilation of peripheral vessels A. Violation of a conductivity in the
D. Occlusion of coronary vessels right auricle
E. Increase of myocardial needs in B. Violation of the automatism of a
oxygen atrio-ventricular node
C. Block of a conductivity in the
385. The calcium canals of atrio-ventricular node
cardiomyocytes have been blocked on D. Arrhythmia
an isolated rabbit's heart. What E. Violation of the automatism of a
changes in the heart's activity can Kiss-Fleck node
happen as a result?
A. Decreased force of the 389. Dystrophic alterations of heart
contraction are accompanied with dilation of heart
B. Decreased heart beat rate cavities, decreased force of heart
C. Decreased rate and force of contractions, increased blood volume
heart beat that remains during systole in the heart
D. Heart stops in diastole cavity, vein overfill. What heart
E. Heart stops in systole condition is it typical for?
A. Emergency stage of
386. A patient in three weeks after hyperfunction and hypertrophy
acute myocardial infarction has pain in B. Cardiosclerosis
the heart and joints and pneumonia. C. Myogenic dilatation
What is the main mechanism of D. Tonogenic dilatation
development of post-infarction E. Cardiac tamponade
Dressler’s syndrome?
A. Autoimmune inflammation 390. A patient who suffers from
B. Vessels ' thrombosis severe disorder of water-salt
C. Secondary infection metabolism experienced cardiac arrest
D. Ischemia of myocardium in diastole. What is the most probable

58
mechanism of cardiac arrest in his breastbone irradiating to his left
diastole? arm. 15 minutes later his condition
A. Hypokaliemia came to normal. Which of the possible
B. Hypernatremia mechanisms of stenocardia
C. Hyperkaliemia development is the leading in this
D. Hyponatremia case?
E. Organism dehydratation A. Functional heart overload
B. High catecholamine concentration
391. While preparing a patient to in blood
the operation the heart chambers' C. Intravascular aggregation of
pressure was measured. In one of blood corpuscles
them the pressure changed during D. Coronary thrombosis
one heart cycle from 0 to 120 mm Hg. E. Coronary atherosclerosis
What chamber of heart was it?
A. Left ventricle 395. In course of a preventive
B. Right atrium examination of a miner a doctor
C. Right ventricle revealed changes of cardiovascular
D. Left atrium fitness which was indicative of cardiac
E. – insufficiency at the compensation
stage. What is the main proof of cardiac
392. ECG of a patient with compensation?
hyperfunction of thyroid gland showed A. Cyanosis
heart hurry. It is indicated by B. Myocardium hypertrophy
depression of the following ECG C. Dyspnea
element: D. Tachycardia
A. QRS complex E. Rise of arterial pressure
B. P-T interval
C. P-Q interval 396. Person has stable HR, not
D. R-R interval more than 40 bpm. What is the
E. P-Q segment pacemaker of the heart rhythm in this
person?
393. A patient ill with essential A. His' bundle
arterial hypertension had a B. Branches of His' bundle
hypertensic crisis that resulted in an C. Purkinye' fibers
attack of cardiac asthma. What is the D. Atrioventricular node
leading mechanism of cardiac E. Sinoatrial node
insufficiency in this case?
A. Blood supply disturbance 397. On examination of the person it
B. Heart overload caused by high was revealed that minute volume of
pressure heart is 3500mL, systolic volume is 50
C. Heart overload caused by mL. What is the frequency of cardiac
increased blood volume contraction?
D. Myocardium damage A. 70 bpm
E. Absolute coronary insufficiency B. 60 bpm
C. 50 bpm
394. A 59 year old patient is a plant D. 80 bpm
manager. After the tax inspection of E. 90 bpm
his plant he felt intense pain behind

59
398. The process of heart the patient's blood. What is the most
transplantation determined the viability likely pathology?
of myocardial cells. The determination A. Rheumatism
of what myocardium parameter is the B. Pancreatitis
most important? C. Cholecystitis
A. Concentration of Ca-ions in D. Miocardial infarction
heart vessels E. Hepatitis
B. Heart temperature
C. Concentration of calcium-ions in 401. A patient presents high activity
myofibrils of LDH1,2, aspartate aminotransferase,
D. Concentration of oxygen in heart creatine phosphokinase. In what organ
vessels (organs) is the development of a
E. Rest potential of cardiomyocytes pathological process the most
probable?
399. Extensive thromboembolic A. In the heart muscle (initial stage
infarction of the left cerebral of myocardium infarction)
hemispheres, large septic spleen, B. In skeletal muscles (dystrophy,
immunocomplex glomerulonephritis, atrophy)
ulcers on the edges of the aortic C. In kidneys and adrenals
valves, covered with polypous D. In liver and kidneys
thrombus with colonies of E. In connective tissue
staphylococcus were revealed on
autopsy of the young man who died in 402. A peripheral segment of vagus
coma. What disease caused cerebral nerve on a dogs neck was being
thromboemboly? stimulated in course of an experiment.
A. Septicemia The following changes of cardiac
B. Acute rheumatic valvulitis activity could be meanwhile observed:
C. Septicopyemia A. Heart rate fall
D. Rheumatic thromboendocarditis B. Increased excitability of
E. Septic bacterial endocarditis myocardium
C. Heart hurry
400. On experiment on the dog the D. Heart rate and heart force
peripheral part of nervus vagus of the amplification
neck was irritated. What changes of E. Enhancement of atrioventricular
the heart function would be observed? conduction
A. Increased atrioventricular
conduction 403. A patient who suffers from
B. Increased contraction force and acute myocarditis has clinical signs of
rate cardiogenic shock. What of the under-
C. Decreased contraction rate mentioned pathogenetic mechanisms
D. Increased contraction force plays the main part in shock
E. Increased myocardial excitability development?
A. Reduction of diastolic flow to the
401. Marked increase of activity of heart
МВ-forms of CPK B. Increase of peripheral vascular
(creatinephosphokinase) and LDH-1 resistance
were revealed on the examination of C. Depositing of blood in organs

60
D. Disturbance of pumping ability of caused by deceleration in ventricles of:
heart A. Depolarization and repolarization
E. Decrease of vascular tone B. Contraction
C. Depolarization
404. A 49 y.o. woman consulted a D. Relaxation
doctor about heightened fatigue and E. Repolarization
dyspnea during physical activity. ECG:
heart rate is 50/min, PQ is extended, 408. Vagus branches that innervate
QRS is unchanged, P wave quanity heart are being stimulated during an
exceeds quantity of QRS complexes. experiment. This caused reduction of
What type of arrhythmia does the heart rate due to the intensification of
patient have? the following process (through the cell
A. Extrasystole membrane of cardiac pacemaker):
B. Ciliary arhythmia A. Calcium ion yield
C. Sinus bradycardia B. Calcium and potassium ion yield
D. Atrioventricular block C. Calcium ion entry
E. Sinoatrial block D. Potassium ion entry
E. Potassium ion yield
405. A patient suffering from
stenocardia was taking nitroglycerine 409. An isolated cell of human heart
which caused restoration of blood automatically generates excitement
supply of myocardium and relieved impulses with frequency of 60 times per
pain in the cardiac area. What minute. This cell was taken from the
intracellular mechanism provides following heart structure:
restoration of energy supply of A. Atrioventricular node
insulted cells? B. Atrium
A. Intensification of RNA C. Ventricle
generation D. Sinoatrial node
B. Intensification of ATP E. His' bundle
resynthesis
C. Intensification of oxygen 410. A 45 year old patient was
transporting into the cell admitted to the cardiological
D. Increased permeability of department. ECG data: negative P
membranes wave overlaps QRS complex, diastolic
E. Reduction of ATP resynthesis interval is prolonged after extrasystole.
What type of extrasystole is it?
406. A patient has delayed A. Ventricular
conduction of excitement through the B. Atrial
atrioventricular node. What changes C. Atrioventricular
of ECG will be observed? D. Sinus
A. Prolongation of Q-T interval E. Bundle-branch
B. S-T-segment displacement
C. Prolongation of P-Q interval 411. A 63 year old male patient who
D. Negative T wave had been suffering from chronic diffuse
E. Prolongation of Q-S interval obstructive disease, pulmonary
emphysema, for 15 years died from
407. ECG of a patient shows cardiac insufficiency. Autopsy revealed
prolongation of T-wav E. This is nutmeg liver cirrhosis, cyanotic

61
induration of kidneys and spleen, C. Heart overload by resistance
ascites, edemata of lower limbs. D. Primary myocardial insufficiency
These changes of internal organs are E. Cardiac tamponade
typical for the following disease:
A. General cardiac insufficiency 415. A 35-year-old man developed
B. Acute left-ventricular acute heart failure while running for a
insufficiency long time. What changes in ionic
C. Acute right-ventricular composition can be observed in the
insufficiency cardiac muscle?
D. Chronic right-ventricular A. Accumulation of Na+ and Ca2+
insufficiency ions in the myocardium cells
E. Chronic left-ventricular B. Reduction of Na+ and Ca2+ ions in
insufficiency the myocardium cells
C. Reduction of K+ and Mg2+ ions in
412. Heart rate of a 30-year-old the extracellular space
man under emotional stress reached D. Accumulation of K+ and Mg2+ ions
112 bpm. The reason for the heart in the myocardium cells
rate increase is the altered condition E. Reduction of Na+ and Ca2+ ions in
of the following conducting system of the extracellular space
heart:
A. Purkinje's fibers 416. The minute blood volume in a
B. Sinoatrial node patient with transplanted heart has
C. His' bundle branches increased as a result of physical
D. Atrioventricular node activity. What regulative mechanism is
E. His' bundle responsible for these changes?
A. Sympathetic conditioned reflexes
413. 12 hours after an accute B. Parasympathetic conditioned
attack of retrosternal pain a patient reflexes
presented a jump of aspartate C. Sympathetic unconditioned
aminotransferase activity in blood reflexes
serum. What pathology is this D. Catecholamines
deviation typical for? E. Parasympathetic unconditioned
A. Myocardium infarction reflexes
B. Diabetes insipidus
C. Collagenosis 417. In response to a change in
D. Diabetes mellitus body position from horizontal to vertical
E. Viral hepatitis blood circulation system develops
reflectory pressor reaction. Which of
414. ECG of a 44-year-old patient the following is its compulsory
shows signs of hypertrophy of both component?
ventricles and the right atrium. The A. Weakening of the pumbing ability
patient was diagnosed with the of heart
tricuspid valve insufficiency. What B. Decrease in the circulating blood
pathogenetic variant of cardiac volume
dysfunction is usually observed in C. Increase in the heart rate
case of such insufficiency? D. Systemic dilatation of the arterial
A. Heart overload by volume resistive vessels
B. Coronary insufficiency

62
E. Systemic constriction of the attack of cardiac asthma. What is the
venous vessels leading mechanism of cardiac
insufficiency in this case?
418. During fighting a man had a A. Blood supply disturbance
cardiac arrest as a result of a hard B. Heart overload caused by high
blow to the upper region of anterior pressure
abdominal wall. Which of the C. Heart overload caused by
described mechanisms might have increased blood volume
provoked the cardiac arrest? D. Myocardium damage
A. Sympathetic unconditioned E. Absolute coronary insufficiency
reflexes
B. Peripheric reflexes 422. Arterial pressure of a surgeon
C. Sympathetic conditioned who performed a long operation rised
reflexes up to 140/110 mm Hg. What changes
D. Parasympathetic unconditioned of humoral regulation could have
reflexes caused the rise of arterial pressure in
E. Parasympathetic conditioned this case?
reflexes A. Activation of kallikrein kinin
system
419. A patient with coronary B. Activation of sympathoadrenal
disease has been diagnosed with system
myocardial hypertrophy, tachycardia C. Inhibition of sympathoadrenal
and a decrease in minute blood system
volume. What is the leading D. Activation of formation and
mechanism of cardiac hystiocyte excretion of aldosterone
damage in this case? E. Activation of renin angiotensive
A. Increase in and system
adrenoreceptors quantity
B. Mg2+ loss by cardiac hystiocytes 423. An adult man presents with
C. Damage of specific membrane systemic arterial pressure drop from
pumps 120/70 to 90/50 mm Hg. This resulted
D. Cardiac hystiocyte dehydration in reflex vasoconstriction.
E. Ca2+ loss by cardiac hystiocytes Vasoconstriction will be minimal in the
following organ:
420. Arterial hypertention is caused A. Bowels
by the stenosis of the renal arteries in B. Liver
the patient. Activation of what system C. Heart
is the main link in the pathogenesys of D. Skeletal muscles
this form of hypertension? E. Skin
A. Parasympathetic
B. Kallikrein-kinin 424. An aged man had raise of
C. Renin-angiotensin arterial pressure under a stress. It was
D. Sympathoadrenal caused by activation of:
E. Hypothalamic-pituitary A. Sympathoadrenal system
B. Functions of thyroid gland
421. A patient ill with essential C. Hypophysis function
arterial hypertension had a D. Functions of adrenal cortex
hypertensic crisis that resulted in an

63
E. Parasympathetic nucleus of E. Stimulation of gastrin production
vagus by G-cells

23. Disorders of 428. A patient complains of frequent


diarrheas, especially after consumption
Digestive System of fattening food, and of body weight
loss. Laboratory examination revealed
425. A 57-year-old patient was steatorrhea; hypocholic feces. What
admitted to the gastroenterological can be the cause of this condition?
department with suspicion on A. Mucous membrane inflammation
Zollinger-Ellison syndrom because of of small intestine
rapid increase of gastrin level in the B. Obturation of biliary tracts
blood serum. What disorder of the C. Lack of pancreatic phospholipase
secretory function of the stomach is D. Lack of pancreatic lipase
the most likely? E. Unbalanced diet
A. Hypoacid hyposecretion
B. Hypoacid hypersecretion 429. Secretion of which
C. Hyperacid hypersecretion gastrointestinal hormones is primerily
D. Hyperacid hyposecretion decreased in patient with removed
E. Achylia duodenum?
A. Gastrin
426. After the usage of B. Histamine
acetylsalicylic acid a patient C. Gastrin and histamine
developed epigastric pain because of D. Cholecystokinin and secretin
exacerbation of his ulcer. What are E. Neurotensin
the principles of this medication
ulcerogenity? 430. During histological examination
A. Immunodepressive effect of the stomach it was found out that
B. Cholagogic effect glands contain very small amount of
C. Antiprostaglandinic effect pariental cells or they are totally absent.
D. Stimulation of pepsin secretion Mucose membrane of what part of the
E. Spasm of vessels stomach was studied?
A. Cardiak part
427. A patient with hypersecretion B. Body of stomach
of the gastric juices was recomended C. Pyloric part
to exclude concentrated bouillons and D. Fundus of stomach
vegetable decoctions from the diet E. -
because of their stimulation of gastric
secretion. What is dominating 431. Profuse foam appeared when
mechanism of stimulation of secretion dentist put hydrogen peroxide on the
in this case? mucous of the oral cavity. What
A. Irritation of mechanoreceptors of enzyme caused such activity?
the oral cavity A. Acetyltransferase
B. Irritation of taste receptors B. Glucose-6-
C. Stimulation of excretion of phosphatdehydrogenase
secretin in the duodenum C. Methemoglobinreductase
D. Irritation of mechanoreceptors of D. Catalase
the stomach E. Cholinesterase

64
enzymes of pancreatic juice. It is also
432. A mother of a newborn accompanied by increase of rate of the
complains of her baby's constant following enzyme:
belching with undigested milk. Which A. Pepsin
developmental anomaly is it an B. Gastricsin
evidence of? C. Renin
A. Anal atresia D. Tripsin
B. Labium leporium E. Enterokinase
C. Esophageal atresia
D. Faux lupinum 437. A 35 year old man consulted a
E. Esophageal fistula dentist about reduced density of dental
tissue, high fragility of teeth during
433. A 30-year-old woman was eating solid food. This patient suffers
diagnosed with insufficiency of the most probably from the deficiency
exocrinous function of pancreas. of the following mineral element:
Hydrolisis of what nutrients will be A. Iron
disturbed? B. Potassium
A. Proteins C. Magnesium
B. Fats, carbohydrates D. Sodium
C. Proteins, carbohydrates E. Calcium
D. Proteins, fats
E. Proteins, fats, carbohydrates 438. A patient died from acute
cardiac insufficiency, among clinical
434. Examination of a 43 y.o. presentations there was gastrointestinal
patient revealed that his stomach has haemorrhage. Examination of mucous
difficulties with digestion of protein membrane of sromach revealed some
food. Gastric juice analysis revealed defects reaching myenteron; their
low acidity. Function of which gastric edges and bottom were mostly even
cells is disturbed in this case? and loose, some of them contained
A. Endocrinous cells dark-red blood. What pathological
B. Parietal exocrinocytes process was revealed?
C. Mucous cells (mucocytes) A. Thrombosis
D. Main exocrinocytes B. Inflammation
E. Cervical mucocytes C. Erosions
D. Chronic ulcers
435. A patient has a disturbed E. Acute ulcers
absorbtion of fat hydrolysates. It might
have been caused by a deficit in the 439. A patient underwent an
small intestine cavity: operation on account of gall bladder
A. Of bile acids excision that resulted in obstruction of
B. Of bile pigments Ca absorption through the bowels wall.
C. Of lipolytic enzymes What vitamin wil stimulate this
D. Of sodium ions process?
E. Of liposoluble vitamins A. D3
B. C
436. A patient consumed a lot of C. B12
reach in proteins food that caused D. K
increase of rate of proteolytic E. PP

65
A. Vitamin K
440. A male patient has been B. Vitamin C
diagnosed with gastric ulcer. C. Pantothenic acid
Bacteriological examination of biopsy D. Vitamin U
material from the affected part of E. Vitamin B1
stomach revealed small colonies of
gram-negative, oxide reductase- 444. A coprological survey revealed
positive flexibacteria that grew on the light-colored feces containing drops of
chocolate agar on the fifth day. Which neutral fat. The most likely reason for
of the following microorganisms is the this condition is the disorder of:
most likely causative agent? A. Bile inflow into the bowel
A. Campilobacter fetus B. Pancreatic juice secretion
B. Chlamydia trachomatis C. Intestinal juice secretion
C. Campilobacter jejuni D. Intestinal absorption
D. Mycoplasma hominis E. Gastric juice acidity
E. Helicobacter pylori
445. A 58-year-old man has a
441. A man is being measured clinical picture of acute pancreatitis.
power inputs on an empty stomach, in The increase of what substance in the
the lying position, under conditions of urine from belowmentioned will confirm
physical and psychic rest at a the diagnosis?
comfortable temperature. Power A. Albumin
inputs will reach the maximum at: B. Nonprotein (rest) nitrogen
A. 7-8 a.m. C. Urea
B. 10-12 a.m. D. Uric acid
C. 3-4 a.m. E. Amylase
D. 5-6 p.m.
E. 2-3 p.m. 446. After intake of rich food a
patient feels nausea and sluggishness;
442. A 30-year-old male patient with time there appeared signs of
with acute pancreatitis has been found steatorrhe a. Blood cholesterine
to have a disorder of cavitary protein concentration is 9,2 mmole/l. This
digestion. The reason for such condition was caused by lack of:
condition can be the hyposynthesis A. Chylomicrons
and hyposecretion of the following B. Fatty acids
enzyme: C. Phospholipids
A. Pepsin D. Bile acids
B. Dipeptidase E. Triglycerides
C. Amylase
D. Lipase 24. Liver Pathology
E. Tripsin
447. M-r S presents all signs of the
443. A doctor recommends a
hepatic coma: loss of consciousness,
patient with duodenal ulcer to drink
absence of reflexes, cramps,
cabbage and potato juice after the
convulsion, disorder of heart activity,
therapy course. Which substances
recurrent (periodical) respiration. What
contained in these vegetables help to
heal and prevent the ulcers?

66
are cerebrotoxical substances which examination of the following substance
accumulate in blood under hepar exreted by urine:
A. Autoantibody A. Creatinine
B. IL-1 B. Uric acid
C. Ammonia C. Ammonium salts
D. Ketonic body D. Amino acids
E. Necrosogenic substances E. Hippuric acid

448. A 48 y.o. patient was admitted 452. Fatty of phospholipids is


to the hospital with complaints about disordered due to fat infiltration of the
weakness, irritability, sleep liver. Indicate which of the presented
disturbance. Objectively: skin and substances can enhance the process of
scleras are yellow. In blood: methylation during phospholipids
conjugated bilirubin, cholalemi a. synthesis?
Feces are acholic. Urine is of dark A. Glycerin
colour (bilirubin). What jaundice is it? B. Citrate
A. Mechanic C. Methionine
B. Gilbert's syndrome D. Ascorbic acid
C. Parenchymatous E. Glucose
D. Hemolytic
E. Crigler-Najjar syndrome 453. Obturative jaundice developed
in a 60-year-old patient because of
449. After consumption of rich food malignant tumour of the big papillary of
a patient has nausea and heartburn, the duodenal. Lumen of what
steatorrhea. This condition might be anatomical structure is squeezed with
caused by: tumour?
A. Disturbed phospholipase A. Cystic duct
synthesis B. Hepatopancreatic ampulla
B. Increased lipase secretion C. Common hepatic duct
C. Amylase deficiency D. Right hepatic duct
D. Bile acid deficiency E. Left hepatic duct
E. Disturbed tripsin synthesis
454. A 59-year-old man has signs of
450. A patient has yellow skin the parenchymatous jaundice and
colour, dark urine, dark-yellow feces. portal hypertension. On histological
What substance will have examination of the puncture of the liver
strengthened concentration in the bioptate, it was revealed: beam-lobule
blood serum? structure is affected, part of
A. Biliverdin hepatocytes has signs of fat dystrophy,
B. Unconjugated bilirubin port-portal connective tissue septa with
C. Verdoglobin formation of pseudo-lobules,with
D. Mesobilirubin periportal lympho-macrophage
E. Conjugated bilirubin infiltrations. What is the most probable
diagnosis?
451. A patient is ill with A. Alcohol hepatitis
hepatocirrhosis. State of antitoxic liver B. Chronic hepatosis
function can be characterized by C. Viral hepatitis
D. Liver cirrhosis

67
E. Toxic dystrophy C. Biliverdin
D. Conjugated bilirubin
455. After intake of rich food a E. Unconjugated bilirubin
patient feels nausea and
sluggishness: with time there 459. Blood analysis of a patient with
appeared signs of steatorrhea. Blood jaundice reveals conjugated
cholesterine concentration is 9.2 bilirubinemia, increased concentration
micromole/l. This condition was of bile acids. There is no
caused by lack of: stercobilinogen in urine. What type of
A. Phospholipids jaundice is it?
B. Chylomicrons A. Hepatocellular jaundice
C. Triglycerides B. Parenchymatous jaundice
D. Fatty acids C. Cythemolytic jaundice
E. Bile acids D. Hemolytic jaundice
E. Obstructive jaundice
456. An experimantal animal that
was kept on protein-free diet 460. Jaundice treatment involves
developed fatty liver infiltration, in administration of barbiturates inducing
particular as a result of deficiency of the synthesis of UDP-glucuronyl
methylating agents. This is caused by transferase. A medicinal effect is
disturbed generation of the following caused by the production of:
metabolite: A. Biliverdin
A. Acetoacetate B. Protoporphyrin
B. Cholesterol C. Heme
C. DOPA D. Indirect reacting (unconjugated)
D. Choline bilirubin
E. Linoleic acid E. Direct reacting (conjugated)
bilirubin
457. Hepatitis has led to the
development of hepatic failure. 461. The greater amount of nitrogen
Mechanism of edemata formation is is excreted from the organism in form of
activated by the impairment of the urea. Inhibition of urea synthesis and
following liver function: accumulation of ammonia in blood and
A. Chologenetic tissues are induced by the decreased
B. Barrier activity of the following liver enzyme:
C. Protein-synthetic A. Urease
D. Antitoxic B. Aspartate aminotransferase
E. Glycogen-synthetic C. Carbamoyl phosphate synthetase
D. Pepsin
458. A 46 year old woman suffering E. Amylase
from chololithiasis developed
jaundice. Her urine became dark- 462. A patient presents with
yellow and feces became colourless. icteritiousness of skin, scleras and
Blood serum will have the highest mucous membranes. Blood plasma the
concentration of the following total bilirubin is increased, stercobilin is
substance: increased in feces, urobilin is increased
A. Mesobilirubin in urine. What type of jaundice is it?
B. Urobilinogen A. Obturational

68
B. Cholestatic 466. A 16 year-old patient got
C. Gilbert's disease numerous traumas in automobile
D. Haemolytic accident. Now the patient is haning a
E. Parenchymatous shock. АP - 80/60 mm Hg. daily urine
volume 60-80 ml. What pathogenic
25. Kidneys Pathology mechanism leads to kidneys function
violation?
A. Increased vasopressin blood
463. Shock and signs of acute
concentration
renal failure (ARF) developed in the
B. Increased osmotic pressure in
patient due to permanent injury. What
glomerular capillaries
is the leading cause of development of
C. Increased pressure in Bowman’s
ARF in the case?
capsule
A. Urine excretion violation
D. Trauma of the urinary bladder
B. Increased pressure in the renal
E. Decreased hydrostatic pressure
arteries
in glomerular capillaries
C. Decreased arterial pressure
D. Decreased oncotic BP
467. Chronic glomerulonephritis was
E. Increased pressure in the
diagnosed in a 34-year-old patient 3
nephron capsule
years ago. Edema has developed
within the last 6 monthes. What caused
464. A 65-year-old suffering from
the edema?
the gout man complains of the pain in
A. Hyperproduction of vasopressin
the kidney's region. On ultrasonic
B. Hyperaldosteronism
examination the renal calculi were
C. Proteinuria
revealed. As a result of what process
D. Liver disfunction of protein
were they formed?
formation
A. Ornithine cycle
E. Hyperosmolarity of plasma
B. Heme decay
C. Decay of purine nucleotides
468. The low specific gravity of the
D. Protein catabolism
secondary urine (1002) was found out
E. Restoration of cysteine
in the sick person. Wat is the most
distant part of nephron where
465. Glomerular filtration rate
concentration of secondary urine takes
(GFR) increased by 20% due to
place?
prolonged starvation of the person.
A. In ascending part of loop of Henle
The most evident cause of filtration
B. In the collecting duck
changes under this conditions is:
C. In the nephron’s glomerulus
A. Increase of filtration coefficient
D. In distal tubule of nephron
B. Increase of penetration of the
E. In proximal tubule of nephron
renal filter
C. Decrease of oncotic pressure of
469. A 58-year-old patient with acute
blood plasma
cardiac insufficiency has decreased
D. Increase of systemic blood
volume of daily urine - oliguria. What is
pressure
the mechanism of this phenomenon?
E. Increase of renal plasma stream
A. Rise of hydrostatic blood pressure
in capillars
B. Decreased glomerular filtration

69
C. Reduced permeamility of renal hyperlipidemia. What condition is the
filter set of these symptoms typical for?
D. Decreased number of A. Chronic renal failure
functioning glomerules B. Acute renal failure
E. Drop of oncotic blood pressure C. Nephrotic syndrome
D. Chronic pyelonephritis
470. Examination of a 43 y.o. E. Nephritic syndrome
anephric patient revealed anemia
symptoms. What is the cause of these 474. As a result of long-term
symptoms? starvation the glomerular filtration of a
A. Folic acid deficit man was accelerated by 20%. The
B. Vitamin B12 deficit most probable cause of filtration
C. Reduced synthesis of changes under such conditions is:
erythropoietins A. Rise of systemic arterial pressure
D. Enhanced destruction of B. Growth of filtration coefficient
erythrocytes C. Increase of renal plasma flow
E. Iron deficit D. Increased permeability of renal
filter
471. Violation of safety rules E. Fall of oncotic pressure of blood
resulted in calomel intoxication. Two plasma
days later the daily diuresis was 620
ml. A patient experienced headache, 475. For a long time a 49-year-old
vomiting, convulsions, dyspnea, moist woman was suffering from
rales in lungs. What pathology is it? glomerulonephritis which caused death.
A. Chronic renal insufficiency On autopsy it was revealed that
B. Acute renal insufficiency kidneys size was 7х3х2.5 sm, weight is
C. Glomerulonephritis 65,0 g, they are dence and small-
D. Uraemic coma grained. Microscopically: fibrinogenous
E. Pyelonephritis inflammation of serous and mucous
capsules, dystrophic changes of
472. A driver who got a trauma in a parenchymatous organs, brain edema.
road accident and is shocked has What complication can cause such
reduction of daily urinary output down changes of serous capsules and inner
to 300 ml. What is the main organs?
pathogenetic factor of such diuresis A. Anemia
change? B. Sepsis
A. Secondary hyperaldosteronism C. DIC-syndrome
B. Drop of arterial pressure D. Thrombopenia
C. Increased vascular permeability E. Uraemia
D. Decreased number of
functioning glomerules 476. Patient complains of frequent
E. Drop of oncotic blood pressure and difficult urination. Imperfection of
what formation can cause it?
473. A 30 year old woman has face A. Bulb-uretic glands
edemata. Examination revealed B. Testicle adnexa
proteinuria (5,87 g/l), C. Prostate
hypoproteinemia, dysproteinemia, D. Testicles
E. Sperm bubbles

70
B. Acceleration of glomerular
477. Periodic renal colics attackes filtration
are observed in the woman with C. Reduced tubular reabsorption of
primery hyperparathyroidizm. Na+ ions
Ultrasonic examination revealed small D. Reduced tubular reabsorption of
stones in the kidneys. What is the water
cause of the formation of the stones? E. Reduced glucose reabsorption
A. Hypercholesterinemia
B. Hyperuricemia 481. Atria of an experimental animal
C. Hyperkalemia were superdistended by blood that
D. Hypercalcemia resulted in decreased reabsorption of
E. Hyperphosphatemia Na+ and water in renal tubules. This
can be explained by the influence of the
478. A 35 y.o. patient who often following factor upon kidneys:
consumes alcohol was treated with A. Renin
diuretics. There appeared serious B. Vasopressin
muscle and heart weakness, vomiting, C. Angiotensin
diarrhea. AP-100/60 mm Hg. D. Aldosterone
depression. This condition is caused E. Natriuretic hormone
by intensified excretion with urine of:
A. Phosphates 482. Examination of a patient
B. Chlorine suffering from cancer of urinary bladder
C. Sodium revealed high rate of serotonin and
D. Calcium hydroxyanthranilic acid. It is caused by
E. Potassium excess of the following amino acid in
the organism:
479. On the 6th day of treatment a A. Alanine
patient with acute renal insufficiency B. Tyrosine
developed polyuria. Diuresis C. Tryptophan
intensification at the beginning of D. Methionine
poliuria stage of acute renal E. Histidine
insufficiency is caused by:
A. Growth of natriuretic factor 483. A concentrated solution of
B. Volume expansion of circulating sodium chloride was intravenously
blood injected to an animal. This caused
C. Reduction of vasopressin decreased reabsorption of sodium ions
content in plasma in the renal tubules. It is the result of
D. Renewal of filtration in nephrons the following changes of hormonal
E. Reduction of aldosteron content secretion:
in plasma A. Reduction of atrial natriuretic
factor
480. A patient has a decreased B. Aldosterone increase
vasopressin synthesis that causes C. Aldosterone reduction
polyuria and as a result of it evident D. Vasopressin increase
organism dehydratation. What is the E. Vasopressin reduction
mechanism of polyuria development?
A. Reduced tubular reabsorption of 484. A patient complains of polyuria
protein (7 liters per day) and polydipsia.

71
Examination reveals no disorders of E. Aldosterone increase
carbohydrate metabolism. These
abnormalities might be caused by the 488. A 38-year-old male patient has
dysfunction of the following endocrine been ill with systemic lupus
gland: erythematosus for three years. He was
A. Adenohypophysis diagnosed with diffuse renal affection
B. Adrenal cortex accompanied by massive edemata and
C. Islets of Langerhans (pancreatic expressive proteinuria. What is the
islets) most likely cause of proteinuria
D. Neurohypophysis development?
E. Adrenal medulla A. Aseptic renal affection
B. Urinary bladder inflammation
485. A patient with massive burns C. Urinary tracts inflammation
developed acute renal insufficiency D. Autoimmune renal affection
characterized by a significant and E. Ischemic renal affection
rapid deceleration of glomerular
filtration. What is the mechanism of its 26. Disorders of
development?
A. Reduction of functioning Endocrine System
nephron number
B. Rise of pressure of tubular fluid 489. A 2-year-old child
C. Reduction of renal blood flow experienced convulsions because of
D. Renal artery embolism lowering calcium ions concentration in
E. Damage of glomerular filter the blood plasma. Function of what
structure is decreased?
486. A 36-year-old female patient A. Thymus
has a history of collagen disease. B. Pineal gland
Urine analysis is likely to reveal an C. Parathyroid glands
increased concentration of the D. Adrenal cortex
following metabolite: E. Hypophysis
A. Urea
B. Indican 490. A 50-year-old patient complains
C. Creatinine of thirst, drinking of a lot of water,
D. Oxyproline marked polyuria. Blood glucose is
E. Urobilinogen 4,8mmol/L, urine glucose and acetone
bodies are absent, urine is colorless,
487. A concentrated solution of specific gravity is 1,002-1,004. What is
sodium chloride was intravenously the cause of polyuria?
injected to an animal. This caused A. Aldosteronism
decreased reabsorption of sodium B. Thyrotoxicosis
ions in the renal tubules. It is the result C. Vasopressin insufficiency
of the following changes of hormonal D. Insulin insufficiency
secretion: E. Hypothyroidism
A. Reduction of atrial natriuretic
factor 491. Arterial hypertension,
B. Vasopressin reduction hyperglycemia, glucosuria were
C. Vasopressin increase observed clinically for a long time in the
D. Aldosterone reduction patient with upper type of obesity.

72
Death was due to the cerebral B. Insulin-dependent diabetes
haemorrhage. Basophilic hypophysis mellitus
adenoma, hyperplasia of adrenal C. Type I glycogenosis
gland cortex were revealed on D. Myxoedema
pathomorphological examination. E. Addison's disease
What is the likely diagnosis?
A. Cushing disease 495. A patient complaining of
B. Diabetes mellitus weight loss (10 kg during 2 months),
C. Acromegaly pallpitation and exophthalmos came to
D. Hypophysis nanism the endocrinologist. For the
E. Adiposogenitalis dystrophy hyperfunction of what endocrine gland
(glands) are these complaints the most
492. The person has decreased typical?
diuresis, hypernatremia, hypokalemia. A. Pancreas
Hypersecretion of what hormon can B. Ovaria
cause such changes? C. Parathyroid glands
A. Vasopressin D. Adrenal glands
B. Adrenalin E. Thyroid
C. Aldosterone
D. Auricular sodiumuretic factor 496. Careless student
E. Parathormone occasionaly met his dean. The
concentration of what hormone will
493. Increased production of most likely increase in the blood of the
thyroidal hormones T3 and T4, weight student?
loss, tachycardia, psychic excitement A. Thyrotropin-releasing hormone
and so on present on thyrotoxicosis. B. Cortisol
How do thyroidal hormones effect C. Somatotropin
energy metabolism in the D. Corticotropin
mitochondrion of cells? E. Adrenalin
A. Stops phosphorylation of
substance 497. A 26-year-old woman is
B. Activates phosphorylation of complaining of thirst and dryness in her
substance mouth. The examination has revealed
C. Disconnect oxidation and glucosuria and blood glucose content of
oxidated phosphorylation 6,5 mmol/l. What condition are these
D. Stops respiratory chain symptoms the most typical for?
E. Activates oxidated A. Diabetes insipidus
phosphorylation B. Steroid diabetes
C. Diabetes mellitus
494. The patient with complaints of D. Renal diabetes
permanent thirst applied to the doctor. E. Alimentary glucosuria
Hyperglycemia, polyuria and
increased concentration of 17- 498. A man because of 1,5 litre
ketosteroids in the urine were blood loss has suddenly reduced
revealed. What disease is the most diuresis. The increased secretion of
likely? what hormone caused such diuresis
A. Steroid diabetes alteration?
A. Parathormone

73
B. Vasopressin
C. Natriuretic 502. A 19-year-old female suffers
D. Corticotropin from tachycardia in rest condition,
E. Cortisol weight loss, excessive sweating,
exophtalmos and irritability. What
499. Some diseases reveal hormone would you expect to find
symptoms of aldosteronism with elevated in her serum?
hypertension and edema due to A. ACTH
sodium retention in the organism. B. Mineralocorticoids
What organ of the internal secretion is C. Cortisol
affected on aldosteronism? D. Insulin
A. Hypophysis E. Thyroxine
B. Testicle
C. Ovaries 503. A patient had been taking
D. Pancreas glucocorticoids for a long time. When
E. Adrenal glands the preparation was withdrawn he
developed the symptoms of disease
500. There is only one hormone aggravation, decreased blood pressure
among the neurohormones which and weakness. What is the reason of
refers to the derivatives of amino this condition?
acids according to classification. Point A. Hyperproduction of ACTH
it out: B. Appearance of adrenal
A. Melatonin insufficiency
B. Vasopressin C. Sensibilization
C. Oxytocin D. Habituation
D. Thyroliberin E. Cumulation
E. Somatotropin
504. A patient suffering from
501. A 46 year-old patient has thyrotoxicosis symptoms of
complained of headache, fatigue, vegetoasthenic syndrome was
thirst, pains in the spine and joints for revealed. What of the following would
the last 2 years. Clinically observed show the histological appearance of a
disproportional enlargement of hands, thyroid gland being stimulated by
feet, nose, superciliary arches. He thyroid-stimulating hormone (TSH)?
notes that he needed to buy bigger A. Increased numbers of
shoes three times. What is the main parafollicular cells
reason of such disproportional B. Decreased numbers of follicular
enlargement of different parts of the cells
body? C. Decreased numbers of
A. Increased sensitivity of the parafollicular capillaries
tissues to growth hormone D. An abundance of colloid in the
B. Joints dystrophy development lumen of the follicle
C. Joints chronic inflammation E. Columnar-shaped follicular cells
development
D. Increased sensitivity of the 505. A 40-year-old patient complains
tissues to insulin of intensive heartbeats, sweating,
E. Cartilaginous tissue proliferation nausea, visual impairment, arm tremor,
under growth hormone influence hypertension. From his anamnesis: 2

74
years ago he was diagnosed with resorbtion of calcium ions and
pheochromocytoma. Hyperproduction decreased resorbtion of phosphate
of what hormones causes the given ions. What hormone causes this
pathology? phenomenon?
A. Aldosterone A. Hormonal form D3
B. Catecholamines B. Parathormone
C. Glucocorticoids C. Aldosterone
D. Thyroidal hormones D. Thyrocalcitonin
E. ACTH E. Vasopressin

506. Thyrotoxicosis leads to 509. A patient with infectious


increased production of thyroidal mononucleosis had been taking
hormones T3 and T4, weight loss, glucocorticoids for two weeks. He was
tachycardia, psychic excitement and brought into remission, but he fell ill
so on. How do thyroidal hormones with acute attack of chronic tonsillitis.
effect energy metabolism in the What action of glucocorticoids caused
mitochondrion of cells? this complication?
A. Activate oxidative A. Immunosuppressive
phosphorylation B. Antiinflammatory
B. Disconnect oxidation and C. Antishock
oxidative phosphorylation D. Antiallergic
C. Stop substrate phosphorylation E. Antitoxic
D. Activate substrate
phosphorylation 510. Intake of oral contraceptives
E. Stop respiratory chain containing sex hormones inhibits
secretion of the hypophysiae
507. A 44-year-old woman hormones. Secretion of which of the
complains of common weakness, indicated hormones is inhibited while
heart pain, considerable increase of taking oral contraceptives with sex
body weigt. Objectively: moon-like hormones?
face, hirsutism, AP- 165/100 mm Hg, A. Thyrotropic
height - 164 cm, weight - 103 kg; fat is B. Follicle-stimulating
mostly accumulated in the region of C. Somatolropic
neck, upper shoulder girdle, stomach. D. Oxytocin
What is the main pathogenetic E. Vasopressin
mechanism of obesity?
A. Decreased production of 511. A girl is diagnosed with
glucagon adrenogenital syndrome
B. Increased production of (pseudohermaphroditism). This
glucocorticoids pathology was caused by
C. Increased production of hypersecretion of the following adrenal
mineralocorticoids hormone:
D. Increased production of insulin A. Androgen
E. Decreased production of B. Estrogen
thyroidal hormones C. Aldosterone
D. Adrenalin
508. Kidneys of a man under E. Cortisol
examination show increased

75
512. Rats being under stress have A. Aldosterone increase
muscular hypertonia and high arterial B. Vasopressin increase
pressure, high glucose concentration C. Aldosterone reduction
in blood and intensified secretion of D. Vasopressin reduction
corticotropin and corticosteroids. In E. Reduction of atrial natriuretic
what stress phase are these animals? factor
A. Terminal
B. Erectile 516. Testosterone and it's analogs
C. Exhaustion increase the mass of skeletal muscles
D. Antishock phase that allows to use them for treatment of
E. Shock phase dystrophy. Due to interaction of the
hormon with what cell substance is this
513. A 59 year old patient is a plant action caused?
manager. After the tax inspection of A. Nuclear receptors
his plant he felt intense pain behind B. Membrane receptors
his breastbone irradiating to his left C. Ribosomes
arm. 15 minutes later his condition D. Chromatin
came to normal. Which of the possible E. Proteins- activators of
mechanisms of stenocardia transcription
development is the leading in this
case? 517. Person felt thirsty after staying
A. Functional heart overload in heat for a long time. Signals of what
B. High catecholamine receptors caused it first of all?
concentration in blood A. Osmoreceptors of the liver
C. Intravascular aggregation of B. Glucoreceptors of hypothalamus
blood corpuscles C. Baroreceptors of aortic arch
D. Coronary thrombosis D. Osmoreceptors of hypothalamus
E. Coronary atherosclerosis E. Sodium receptors of
hypothalamus
514. Parents of a 10 year old boy
consulted a doctor about extension of 518. Patient was on glucocorticoids
hair-covering, growth of beard and for a long time, discontinuation of
moustache, low voice. Intensified usage caused exacerbation of the
secretion of which hormone must be illness, decreased BP, weakness. How
assumed? can you explain it?
A. Of progesterone A. Adaptation to the medicine
B. Of oestrogen B. Sensitization
C. Of cortisol C. Hyperproduction of ACTH
D. Of somatotropin D. Insufficiency of adrenal glands
E. Of testosterone E. Cumulation

515. A concentrated solution of 519. During pubescence the cells of


sodium chloride was intravenously male sexual glands begin to produce
injected to an animal. This caused male sex hormon testosterone that
decreased reabsorption of sodium calls forth secondary sexual characters.
ions in the renal tubules. It is the result What cells of male sexual glands
of the following changes of hormonal produce this hormone?
secretion: A. Spermatozoa

76
B. Leidig cells formations, cysts. Bony tissue was
C. Sustentocytes substituted by fibrous connective
D. Supporting cells tissue. Examination revealed also
E. Sertoli's cells adenoma of parathyroid gland and
hypercalcemia. What is the most
520. Inhabitants of territories with probable diagnosis?
cold climate have high content of an A. Osteopetrosis
adaptive thermoregulatory hormone. B. Parathyroid osteodystrophy
What hormone is meant? C. Paget's disease
A. Glucagon D. Osteomyelitis
B. Insulin E. Myelomatosis
C. Cortisol
D. Thyroxin 524. A 4 year old child with
E. Somatotropin hereditary renal lesion has signs of
rickets, vitamin D concentration in
521. A patient suffering from blood is normal. What is the most
pheochromocytoma complains of probable cause of rickets
thirst, dry mouth, hunger. Blood test development?
for sugar revealed hyperglycemia. A. Impaired synthesis of calcitriol
What type of hyperglycemia is it? B. Lack of calcium in food
A. Adrenal C. Hypofunction of parathyroid
B. Alimentary glands
C. Hypercorticoid D. Hyperfunction of parathyroid
D. Somatotropic glands
E. Hypoinsulinemic E. Increased excretion of calcium

522. Examination of a 42 year old 525. Emotional stress causes


patient revealed a tumour of activation of hormon-sensitive
adenohypophysis. Objectively: the triglyceride lipase in the adipocytes.
patient's weight is 117 kg, he has What secondary mediator takes part in
moon-like hyperemic face, red-blue this process?
striae of skin distension on his belly. A. Diacylglycerol
Osteoporosis and muscle dystrophy B. Ions of Са2+
are present. AP is 210/140 mm Hg. C. Cyclic guanosine monophosphate
What is the most probable diagnosis? D. Cyclic adenosine monophosphate
A. Cushing's syndrome E. Adenosine monophosphate
B. Conn's disease
C. Cushing's disease 526. Cardinal symptoms of primary
D. Essential hypertension hyperparathyroidism are osteoporosis
E. Diabetes mellitus and renal lesion along with
development of urolithiasis. What
523. A 50 year old patient has been substance makes up the basis of these
taking treatment thrice for the last 6 calculi in this disease?
months because of fractures caused A. Uric acid
by domestic accidents. Microscopical B. Bilirubin
examination of bony tissue revealed C. Cystine
foci of lacunar resolution, giant-cell D. Cholesterol
granulomas in the tumour-like E. Calcium phosphate

77
C. Glucagon
527. The secretion of whixh D. Somatotropin
hypophysial hormones will be inhibited E. Thyrocalcitonin
after taking the oral contraceptives
containing sex hormones? 531. A patient has been given high
A. Ocytocin doses of hydrocortisone for a long time.
B. Thyrotrophic hormone This caused atrophy of one of the
C. Vasopressin adrenal cortex zones. Which zone is it?
D. Gonadotropic hormone A. Reticular
E. Somatotropic hormone B. Fascial
C. Glomerular and reticular
528. A female patient presents with D. Glomerular
endocrine dysfunction of follicular cells E. -
of the ovarian follicles resulting from
an inflammation. The synthesis of the 27. Disorders of CNS
following hormone will be inhibited:
A. Lutropin
B. Follistatine 532. A 68-year-old woman can't
C. Estrogen move by the upper and lower right
D. Follicle stimulating hormone extremities due to insult. Muscle tone
E. Progesterone of these extremities and reflexes are
increased. There are pathological
529. A 46-year-old patient suffering reflexes. What form of the paralysis is
from the diffuse toxic goiter underwent it?
resection of the thyroid gland. After A. Hemiplegia
the surgery the patient presents with B. Tetraplegia
appetite loss, dyspepsia, increased C. Monoplegia
neuromuscular excitement. The body D. Dissociation
weight remained unchanged. Body E. Paraplegia
temperature is normal. Which of the
following has caused such a condition 533. Due to cranial trauma the
in this patient? patient developed the symptoms:
A. Reduced production of thyroxin intention tremor, dysmetry,
B. Reduced production of adiadochokinesis, dysarthria. What
parathormone structure of the brain is injured?
C. Increased production of A. Striatum
thyroliberin B. Motor cortex
D. Increased production of C. Cerebellum
calcitonin D. Pale sphere
E. Increased production of thyroxin E. Black substance

530. A 32-year-old patient 534. A patient after hypertension


consulted a doctor about the absence stroke does not have voluntary
of lactation after parturition. Such movements in his right arm and leg with
disorder might be explained by the the increased muscle tone in these
deficit of the following hormone: extremites. What type of disfunction of
A. Prolactin nervous system is it?
B. Vasopressin A. Reflex paresis

78
B. Peripheral paresis C. Muscle that dilatates pupil
C. Central paresis D. Cornea
D. Central paralysis E. Lens
E. Peripheral paralysis
539. A 50-year-old patient was
535. A student is thoroughly injured on the occipital region of the
summarising a lecture. When his head. The closed skull trauma was
groupmates begin talking the quality diagnosed. She was taken to the
of the summarising worsens greatly. hospital. The medical examination:
What type of inhibition in the cerebral deregulation of walking and balance,
cortex is the cause of it? trembling of arms. What part of brain
A. Dying was injured?
B. Differential A. The mind-brain
C. External B. The inter-brain
D. Protective C. The cerebellum
E. Delayed D. The spinal cord
E. The medulla oblongata
536. A 60-year-old patient was
diagnosed with hypothalamic lateral 540. Microspecimen of spinal cord
nuclei stroke. What changes in contains a nucleus that should be
patient’s behavior may be expected? analyzed. Its neurons form motor
A. Thirst endings in the skeletal muscles. What
B. Aggressive behaviour nucleus of spinal cord is meant?
C. Unsatisfied hunger A. Intermediate lateral nucleus
D. The rejection of food B. Proper nucleus of the posterior
E. Depression horn
C. Proper nucleus of gray substance
537. Inhibition of alpha-motoneuron D. Proper nucleus of the anterior
of the extensor muscles was noticed horn
after stimulation of α-motoneuron of E. Thoracic nucleus
the flexor muscles during the
experiment on the spinal column. 541. When a patient with traumatic
What type of inhibition can be caused impairment of the brain was examined,
by this process? it was discovered that he had stopped
A. Recurrent to distinguish displacement of an object
B. Reciprocal on the skin. What part of the brain was
C. Presynaptic damaged?
D. Depolarizational A. Frontal central gurus
E. Lateral B. Frontal zone
C. Posterior central gurus
538. While shifting the gaze to the D. Parietal zone of the cortex
closely situated object the refracting E. Occipital zone of the cortex
power of eye's optical mediums will
increase by 10 diopters. It results from 542. A 60-year-old patient has
changing of such eye structure: reduced perception of high-frequency
A. Liquid of the anterior chamber of sounds. What structures' disorder of
eye auditory analizer caused these
B. Vitreous body changes?

79
A. Eustachian tube caused by damage of the following
B. Main membrane of cochlea near tract:
helicotrema A. Lateral spinocortical
C. Muscles of middle ear B. Anterior spinocerebellar
D. Tympanic membrane C. Medial spinocortical
E. Main membrane of cochlea near D. Spinothalamic
the oval window E. Posterior spinocerebellar

543. A 60-year-old man after 547. 54 Vagi of an experimental


cerebral hemorrhage felt asleep for a animal were cut on both sides. What
long time. Damage of what structure respiration changes will be observed?
caused this state? A. It will become shallow and
A. Hippocampus infrequent
B. Nuclears of the cerebral nerves B. It will become deep and frequent
C. Cortex of the large hemispheres C. It will become deep and
D. Reticular formation infrequent
E. Black substances D. It will become shallow and
frequent
544. An experimental rat with E. No changes will be observed
extremity paralysis has no tendon and
cutaneous reflexes, muscle tone is 548. A 28 year old man had a
decreased, but muscles of the gunshot wound of shin that resulted in
affected extremity maintain their ability an ulcer from the side of the injury.
to react with excitation to the direct What is the main factor of
action of continious current. What type neurodystrophy pathogenesis in this
of paralysis is it? case?
A. Flaccid peripheral A. Microcirculation disturbance
B. Extrapyramidal B. Tissue damage
C. Spastic central C. Traumatization of peripheral
D. Flaccid central nerve
E. Spastic peripheral D. Infection
E. Psychical stress
545. In course of an experiment a
skeletal muscle is being stimulated by 549. The ventral roots of 5 frontal
a series of electric impulses. What segment of spinal cord were cut during
type of muscle contraction will arise, if experiment in the animal. What
every subsequent impulse comes in changes will take place in the
the period of shortening of the innervation region?
previous single muscle contraction? A. Loss of temperature sensitivity
A. Partial tetanus B. Loss of proprioceptive sensitivity
B. A series of single contractions C. Loss of movements
C. Muscle contracture D. Loss of touch sensitivity
D. Holotetanus E. Hypersensitivity
E. Asynchronous tetanus
550. Vegetative abnormalities in the
546. As a result of spinal-cord sleep, heat regulation, all kinds of
trauma a 33 y.o. man has a disturbed metabolism, diabetes insipidus are
pain and temperature sensitivity that is developing in the patient due to grouth

80
of the tumour in the III ventricle of substances absorbed by the
brain. Irritation of the nucleus of what bloodstream from the intestines was
part of the brain can cause this revealed. What substances that are
symptoms? created in the intestines can cause
A. Hypothalamus endotoxemia?
B. Cerebral peduncles (cruces A. Acetacetate
cerebri) B. Biotin
C. Mesencephalic tegmentum C. Ornithine
D. Pons cerebelli D. Indole
E. Medulla E. Butyrate

551. Middle part of cochlear of 555. Accelerated frequency of the


internal ear was destroyed in animal heart rate and increased blood
while experiment. It will cause pressure were marked in the sportsman
abnormalities of the sound perception on the start before the competitions.
of the following frequencies: Influence of what parts of the CNS can
A. Low explain these changes?
B. High A. Mesencephalon
C. High and low B. Diencephalon
D. No abnormalities C. Hypothalamus
E. Middle D. Cortex of the large hemispheres
E. Medulla
552. Isolated muscle of a frog is
rhythmically irritated with electric 556. As a result of damage to certain
impulses. Every next impulse is in a structures of brainstem an animal lost
period of relaxation from the previus orientation reflexes. What structures
contraction. What contraction of the were damaged?
muscle appears? A. Vestibular nuclei
A. Asynchronous B. Red nuclei
B. Continuous(smooth) tetanus C. Black substance
C. Waved tetanus D. Medial nuclei of reticular
D. Single formation
E. Tonic E. Quadritubercular bodies

553. A patient has a haemorrhage 557. Glutamate decarboxylation


into the posterior central gyrus. What results in formation of inhibitory
type of sensitivity on the opposite side transmitter in CNS. Name it:
will be disturbed? A. Glutathione
A. Auditory B. GABA
B. Skin and proprioceptive C. Serotonin
C. Visual D. Histamine
D. Olfactory E. Asparagine
E. Auditory and visual
558. A 63 y.o. man with collapse
554. Patient with encephalopathy symptoms was delivered to the
was admitted to the neurological in- emergency hospital. A doctor chose
patient department. Correlation of noradrenaline in order to prevent
increasing of encephalopathy and

81
hypotension. What is the action C. Fascicle of Goll and cuneate
mechanism of this medication? fascicle
A. Activation of dopamine receptors D. Rubrospinal tract
B. Block of M-cholinoreceptors E. Reticulospinal tract
C. Activation of serotonin receptors
D. Activation of β-adrenoreceptors 563. A patient got a trauma that
E. Activation of α-adrenoreceptors caused dysfunction of motor centres
regulating activity of head muscles. In
559. Ammonia is a very toxic what parts of cerebral cortex is the
substance, especially for nervous respective centre normally localized?
system. What substance takes the A. Inferior part of precentral gyrus
most active part in ammonia B. Angular gyrus
detoxication in brain tissues? C. Superior parietal lobule
A. Lysine D. Superior part of precentral gyrus
B. Glutamic acid E. Supramarginal gyrus
C. Histidine
D. Proline 564. A patient was diagnosed with
E. Alanine paralysis of facial and masticatory
muscles. The haematoma is inside the
560. A man who went for a ride on genu of internal capsule. What
a roundabout had amplification of conduction tract is damaged?
heart rate, sweating and nausea. A. Tr. cortico-fronto-pontinus
What receptors stimulation is it B. Tr. cortico-thalamicus
primarily connected with? C. Tr. cortico-temporo-parieto-
A. Visual occipito-pontinus
B. Vestibular D. Tr. cortico-spinalis
C. Proprioceptors E. Tr. cortico-nuclearis
D. Auditory
E. Tactors 565. In course of an experiment
thalamocortical tracts of an animal were
561. A patient staggers and walks cut. What type of sensory perception
astraddle. He has hypomyotonia of remained intact?
arm and leg muscles, staccato A. Visual
speech. In what brain section is this B. Olfactory
affection localized? C. Auditory
A. Cerebellum D. Exteroreceptive
B. Red nucleus E. Nociceptive
C. Caudate nucleus
D. Motor cortex 566. An infectious disease caused
E. Putamen contractive activity of muscles that
contract and dilate eye pupil (paralytic
562. A patient got an injury of state). What functional eye system was
spinal marrow in a road accident that damaged?
caused loss of tactile sensation, A. Photosensory
posture sense, vibration sense. What B. Accomodative
conduction tracts are damaged? C. Lacrimal apparatus
A. Anterior spinocerebellar tract D. Dioptric
B. Tectospinal tract E. Ancillary

82
E. Temporal
567. A patient underwent an
extraction of a part of a CNS 571. A 75-year-old-female patient
structures by medical indications. As a with complaints of visual impairment
result of the extraction the patient has been delivered to the
developed atony, astasia, intention ophthalmologic department. Objective
tremor, ataxy and adiadochokinesis. examination revealed a brain tumor in
Which part of CNS structure had been area of the left optic tract. The patient
extracted? has a visual field defect in the following
A. Cerebellum area:
B. Limbic system A. Left half of both eyes retina
C. Basal ganglions B. Left and right halves of both eyes
D. Amygdaloid corpus retina
E. Hippocamp C. Left and right halves of the right
eye retina
568. After a craniocerebral trauma D. Left and right halves of the left
a patient lost the ability to execute eye retina
learned purposeful movements E. Right half of both eyes retina
(apraxia). The injury is most likely
localized in the following region of the 572. A 49-year old female patient
cerebral cortex: has limitation of left limbs arbitrary
A. Gyrus parahippocampalis movements. Muscular tonus of left
B. Gyrus paracentralis hand and leg is overstrained and
C. Gyrus angularis spasmodic, local tendon reflexes are
D. Gyrus lingualis strong, pathological reflexes are
E. Gyrus supramarginalis presented. What is the most likely
development mechanism of
569. A man who is riding the hypertension and hyperreflexia?
carousel presents with increased A. Motoneuron activation induced by
heart rate, sweating, nausea. This stroke
condition is caused primarily by the B. Activation of excitatory influence
stimulation of the following receptors: from the focus of stroke
A. Auditory C. Reduction of descending
B. Vestibular otolithic inhibitory influence
C. Visual D. Activation of synaptic
D. Proprioceptors transmission
E. Vestibular ampullar E. Ihibition of cerebral cortex
motoneurons
570. A man having a hearing loss
after a head trauma was delivered to
the neurosurgery department. The
cause of the hearing loss might be the
damage of the following lobe of
cerebral cortex:
A. Postcentral gyrus
B. Occipital
C. Frontal
D. Parietal

83
84
29. Answers to the Tests
1. E 51. A 101. E 151. A 201. C 251. D 301. C 351. E 401. A 451. E
2. B 52. C 102. A 152. E 202. E 252. A 302. A 352. D 402. A 452. C
3. C 53. B 103. B 153. B 203. C 253. D 303. C 353. D 403. D 453. B
4. C 54. B 104. C 154. D 204. A 254. E 304. D 354. E 404. D 454. D
5. C 55. E 105. D 155. A 205. A 255. A 305. A 355. A 405. B 455. E
6. E 56. B 106. B 156. B 206. C 256. E 306. B 356. A 406. B 456. D
7. B 57. C 107. D 157. C 207. D 257. B 307. A 357. A 407. E 457. C
8. C 58. E 108. E 158. D 208. A 258. B 308. B 358. B 408. E 458. D
9. D 59. C 109. D 159. B 209. E 259. D 309. C 359. D 409. D 459. E
10. D 60. C 110. D 160. D 210. C 260. A 310. E 360. C 410. C 460. E
11. A 61. E 111. A 161. D 211. B 261. E 311. E 361. D 411. D 461. C
12. A 62. D 112. E 162. E 212. A/D 262. D 312. D 362. B 412. B 462. D
13. E 63. B 113. A 163. B 213. C 263. C 313. A 363. E 413. A 463. C
14. E 64. C 114. B 164. E 214. D 264. B 314. E 364. C 414. A 464. C
15. C 65. A 115. C 165. C 215. E 265. A 315. E 365. A 415. A 465. C
16. D 66. A 116. B 166. C 216. C 266. A 316. A 366. B 416. D 466. E
17. A 67. E 117. C 167. E 217. E 267. C 317. A 367. A 417. E 467. C
18. B 68. D 118. D 168. D 218. E 268. D 318. B 368. E 418. D 468. B
19. E 69. A 119. D 169. A 219. C 269. D 319. E 369. B 419. C 469. B
20. A 70. D 120. C 170. E 220. D 270. C 320. A 370. B 420. C 470. C
21. A 71. D 121. A 171. A 221. C 271. D 321. E 371. A 421. B 471. B
22. E 72. E 122. E 172. D 222. C 272. C 322. C 372. E 422. B 472. B
23. A 73. D 123. A 173. C 223. A 273. B 323. D 373. D 423. C 473. C
24. C 74. E 124. D 174. E 224. E 274. E 324. E 374. C 424. A 474. E
25. B 75. E 125. C 175. C 225. E 275. A 325. D 375. E 425. C 475. E
26. C 76. D 126. D 176. B 226. B 276. A 326. D 376. A 426. C 476. C
27. C 77. D 127. E 177. E 227. B 277. A 327. E 377. C 427. E 477. D
28. A 78. B 128. B 178. D 228. B 278. A 328. C 378. E 428. B 478. E
29. B 79. C 129. C 179. B 229. E 279. C 329. C 379. E 429. D 479. D
30. D 80. C 130. D 180. E 230. D 280. A 330. B 380. B 430. C 480. D
31. A 81. E 131. C 181. C 231. D 281. A 331. E 381. B 431. D 481. E
32. A 82. A 132. E 182. D 232. D 282. D 332. D 382. C 432. C 482. C
33. C 83. D 133. B 183. B 233. C 283. A 333. A 383. C 433. E 483. C
34. E 84. E 134. C 184. E 234. A 284. B 334. D 384. B 434. B 484. D
35. B 85. E 135. D 185. C 235. C 285. B 335. A 385. C 435. A 485. C
36. E 86. E 136. C 186. E 236. D 286. D 336. A 386. A 436. D 486. D
37. A 87. B 137. D 187. D 237. D 287. D 337. C 387. D 437. E 487. D
38. E 88. C 138. A 188. D 238. B 288. A 338. A 388. E 438. E 488. D
39. E 89. A 139. A 189. D 239. D 289. D 339. D 389. C 439. A 489. C
40. C 90. A 140. E 190. A 240. B 290. E 340. E 390. C 440. E 490. C
41. A 91. E 141. D 191. D 241. E 291. B 341. E 391. A 441. D 491. A
42. B 92. D 142. A 192. C 242. C 292. B 342. C 392. D 442. E 492. C
43. A 93. E 143. A 193. E 243. C 293. C 343. A 393. B 443. D 493. C
44. E 94. D 144. B 194. E 244. B 294. C 344. E 394. B 444. A 494. A
45. C 95. A 145. D 195. C 245. C 295. C 345. D 395. B 445. E 495. E
46. C 96. A 146. E 196. A 246. B 296. E 346. C 396. D 446. D 496. E
47. C 97. E 147. E 197. C 247. C 297. A 347. A 397. A 447. C 497. D
48. D 98. C 148. C 198. E 248. E 298. C 348. D 398. E 448. A 498. B
49. D 99. A 149. A 199. B 249. E 299. C 349. A 399. E 449. D 499. E
50. D 100. A 150. B 200. A 250. B 300. C 350. C 400. C 450. B 500. A

501. E 511. A 521. A 531. B 541. C 551. E 561. A 571. A


502. E 512. D 522. C 532. A 542. E 552. C 562. C 572. C
503. B 513. B 523. B 533. C 543. D 553. B 563. A 573.
504. E 514. E 524. A 534. D 544. A 554. D 564. E 574.
505. B 515. C 525. D 535. C 545. D 555. D 565. B 575.
506. B 516. A 526. E 536. D 546. D 556. E 566. B 576.
507. B 517. D 527. D 537. B 547. C 557. B 567. A 577.
508. B 518. D 528. C 538. E 548. C 558. E 568. E 578.
509. A 519. B 529. B 539. C 549. C 559. B 569. E 579.
510. B 520. D 530. A 540. D 550. A 560. B 570. E 580.

RECCOMENDED LITERATURE

1. Stephen J. McPhee. Pathophysiology of Disease. An introduction to


Clinical Medicine / Stephen J. McPhee, William F. Ganong // Lunge Medical
Books/McGraw-Hill. – 5th edition. – 2006. – 761 p.
2. Pathophysiology: concepts of altered health states / [edited by] Carol
Mattson Porth, Glenn Matfin. – 8th ed. Copyright © Wolters Kluwer Health I
Lippincott Williams & Wilkins. – 2009. – p. 1688.
3. Pathophysiology-Патофізіологія: підручник / Н.К. Сімеонова; за
науковою редакцією В.А. Міхньова. – К.: ВСВ «Медицина», 2010. – 544 с.
4. General and clinical pathophysiology / Edited by Anatoliy V. Kubyshkin –
Vinnytsia: Nova Knuha Publishers – 2011.
5. Handbook of general and Clinical Pathophysiology / Edited by prof.
A.V.Kubyshkin. – CSMU. – 2005.
6. Pathophysiology / Edited by prof. Zaporozan. – OSMU. – 2005.
7. Essentials of Pathophysiology: Concepts of Altered Health States
(Lippincott Williams & Wilkins), Trade paperback / Carol Mattson Porth, Kathryn J.
Gaspard. – 2003.
8. J.B.Walter I.C.Talbot General pathology. Seventh edition. – 1996.
9. Robbins and Cotran Pathologic Basis of Disease 7th edition / Kumar,
Abbas, Fauto. – 2006.
10. Pathophysiology, Concepts of Altered Health States, Carol Mattson Porth,
Glenn Matfin. – New York, Milwaukee. – 2009.
11. http://testcentr.org.ua/
12. http://kroktest.org.ua/kroki/krok-1/foreign-tests/examination-tests-or-
booklets-foreign-students-englishstep-1
13. http://online.kroktest.org.ua/

86
87
88

You might also like